Test Bank – ACP: Hematology

Test Bank – ACP: Hematology

Approach to Hematologic Disorders

1. A 43-year-old woman presents for the evaluation of bleeding gums. The patient reports that for the past 2 months, her gums have bled more easily when she brushes her teeth. Physical examination reveals palatal petechiae and scattered petechiae over the lower extremities bilaterally.

Which of the following laboratory tests is most likely to identify the abnormality responsible for this patient’s bleeding disorder?
A. Prothrombin time (PT) and international normalized ratio (INR)
B. Complete blood count (CBC)
C. Partial thromboplastin time (PTT)
D. A mixing study

Key Concept/Objective: To understand that thrombocytopenia usually presents as petechial bleeding

Bleeding occurs as a consequence of thrombocytopenia, deficiencies of coagulation factors, or both. Thrombocytopenia usually presents as petechial bleeding that is first observed in the lower extremities. Deficiencies in coagulation factor more often cause bleeding into the gastrointestinal tract or joints. Intracranial bleeding, however, can occur with a deficiency of platelets or coagulation factors and can be catastrophic. CBCs are routinely performed in most laboratories through the use of an electronic particle counter, which determines the total white blood cell and platelet counts and calculates the hematocrit and hemoglobin from the erythrocyte count and the dimensions of the red cells. For this patient, a CBC would likely disclose a decreased platelet count (thrombocytopenia). Impaired hepatic synthetic function and vitamin K deficiency would result in prolongation of the PT and INR. Coagulation factor deficiencies and coagulation factor inhibitors would result in prolongation of the PTT. A mixing study is obtained to differentiate between a coagulation factor deficiency and a coagulation factor inhibitor by mixing patient plasma with normal plasma in the laboratory. (Answer: B—Complete blood count [CBC])

2. A 53-year-old man presents with fatigue, weight loss, and a petechial rash. A CBC reveals anemia and thrombocytopenia, with a peripheral smear containing 20% blast cells. A bone marrow biopsy is performed, revealing acute myelogenous leukemia (AML). The patient is treated with cytarabine and daunorubicin induction chemotherapy.

Which of the following blood cell lineages has the shortest blood half-life and is therefore most likely to become deficient as a result of this patient’s chemotherapy treatment?
A. Red blood cells (RBCs)
B. Platelets
C. Megakaryocytes
D. Neutrophils

Key Concept/Objective: To understand differences in the dynamics of erythrocytes, platelets, and leukocytes in the blood

There are important differences in the dynamics or kinetics of erythrocytes, platelets, and leukocytes in the blood. For instance, neutrophils have a blood half-life of only 6 to 8 hours; essentially, a new blood population of neutrophils is formed every 24 hours. Erythrocytes last the longest by far: the normal life span is about 100 days. These differences partially account for why neutrophils and their precursors are the predominant marrow cells, whereas in the blood, erythrocytes far outnumber neutrophils. Similarly, the short half-life and high turnover rate of neutrophils account for why neutropenia is the most frequent hematologic consequence when bone marrow is damaged by drugs or radiation. Transfusion of erythrocytes and platelets is feasible because of their relatively long life span, whereas the short life span of neutrophils has greatly impeded efforts to develop neutrophil transfusion therapy. (Answer: D—Neutrophils)

3. A 54-year-old man presents with fatigue, weakness, and dyspnea on exertion. Physical examination reveals conjunctival pallor, palatal petechiae, and splenomegaly. A CBC reveals profound anemia and thrombocytopenia. A bone marrow biopsy reveals agnogenic myeloid metaplasia (myelofibrosis). The patient’s splenomegaly is attributed to increased production of blood cells in the spleen.

Which of the following terms indicates blood cell production outside the bone marrow in the spleen, liver, and other locations?
A. Adjunctive hematopoiesis
B. Remote hematopoiesis
C. Accessory hematopoiesis
D. Extramedullary hematopoiesis

Key Concept/Objective: To understand the nomenclature of hematopoiesis

Hematopoiesis begins in the fetal yolk sac and later occurs predominantly in the liver and the spleen. Recent studies demonstrate that islands of hematopoiesis develop in these tissues from hemangioblasts, which are the common progenitors for both hematopoietic and endothelial cells. These islands then involute as the marrow becomes the primary site for blood cell formation by the seventh month of fetal development. Barring serious damage, such as that which occurs with myelofibrosis or radiation injury, the bone marrow remains the site of blood cell formation throughout the rest of life. In childhood, there is active hematopoiesis in the marrow spaces of the central axial skeleton (i.e., the ribs, vertebrae, and pelvis) and the extremities, extending to the wrists, the ankles, and the calvaria. With normal growth and development, hematopoiesis gradually withdraws from the periphery. This change is reversible, however; distal marrow extension can result from intensive stimulation, as occurs with severe hemolytic anemias, long-term administration of hematopoietic growth factors, and hematologic malignancies. The term medullary hematopoiesis refers to the production of blood cells in the bone marrow; the term extramedullary hematopoiesis indicates blood cell production outside the marrow in the spleen, liver, and other locations. (Answer: D—Extramedullary hematopoiesis)

4. A 35-year-old woman with advanced HIV disease complicated by anemia is seen for routine follow-up.
The patient is started on erythropoeitin to decrease the severity of her anemia and to provide symptomatic improvement.

Which of the following laboratory findings is the most easily monitored immediate effect of erythropoietin therapy?
A. An increase in the reticulocyte count
B. An increase in the mean corpuscular volume
C. An increase in the hemoglobin level
D. An increase in the mean corpuscular hemoglobin level

Key Concept/Objective: To understand that an increase in the reticulocyte count is the most easily monitored immediate effect of erythropoietin therapy

Erythropoietin is a glycosylated protein that modulates erythropoiesis by affecting several steps in red cell development. The peritubular interstitial cells located in the inner cortex and outer medulla of the kidney are the primary sites for erythropoietin production. Erythropoietin can be administered intravenously or subcutaneously for the treatment of anemia caused by inadequate endogenous production of erythropoietin. Treatment is maximally effective when the marrow has a generous supply of iron and other nutrients, such as cobalamin and folic acid. For patients with renal failure, who have very low erythropoietin levels, the starting dosage is 50 to 100 units subcutaneously three times a week. The most easily monitored immediate effect of increased endogenous or exogenous erythropoietin is an increase in the blood reticulocyte count. Normally, as red cell precursors mature, the cells extrude their nucleus at the normal blast stage. The resulting reticulocytes, identified by the supravital stain of their residual ribosomes, persist for about 3 days in the marrow and 1 day in the blood. Erythropoietin shortens the transit time through the marrow, leading to an increase in the number and proportion of blood reticulocytes within a few days. In some conditions, particularly chronic inflammatory diseases, the effectiveness of erythropoietin can be predicted from measurement of the serum erythropoietin level by immunoassay. It may be cost-effective to measure the level before initiating treatment in patients with anemia attributable to suppressed erythropoietin production, such as patients with HIV infection, cancer, and chronic inflammatory diseases. Several studies have shown that erythropoietin treatment decreases the severity of anemia and improves the quality of life for these patients. In patients with anemia caused by cancer and cancer chemotherapy, current guidelines recommend erythropoietin treatment if the hemoglobin level is less than 10 g/dl. (Answer: A—An increase in the reticulocyte count)

5. A 55-year-old man with type 1 diabetes undergoes dialysis three times a week for end-stage renal disease.
You recently started him on erythropoietin injections for chronic anemia (hematocrit, 25%).

Which of the following is the best test to determine whether this patient will respond to the erythropoietin treatment?
A. Erythropoietin level
B. Hematocrit
C. Creatinine level
D. Reticulocyte count
E. Blood urea nitrogen

Key Concept/Objective: To understand the site of production, effect, and therapeutic monitoring of erythropoietin

In many renal diseases, the kidneys fail to produce sufficient amounts of erythropoietin. Replacement of endogenous erythropoietin stimulates red cell precursors in the bone marrow to mature more quickly. If the patient has normal bone marrow, an elevated reticulocyte count should be seen several days after initiation of therapy. (Answer: D—Reticulocyte count)

6. A 34-year-old woman undergoes chemotherapy for advanced-stage breast cancer. As expected, she develops pancytopenia.

Which cell line would you expect to be the last to recover in this patient?
A. Eosinophils
B. Platelets
C. Basophils
D. Monocytes
E. RBCs

Key Concept/Objective: To understand the time needed for cell-line recovery after bone marrow damage

The proliferation and maturation of platelets take longer than those of either red blood cells (7 to 10 days) or white blood cells (10 to 14 days) and thus are the slowest to recover from an acute bone marrow injury, such as occurs with chemotherapy. (Answer: B—Platelets)

For more information, see Dale DC: 5 Hematology: I Approach to Hematologic Disorders. ACP Medicine Online (www.acpmedicine.com). Dale DC, Federman DD, Eds. WebMD Inc., New York, June 2003

Red Blood Cell Function and Disorders of Iron Metabolism

7. An 86-year-old man visits your clinic for routine follow-up. Upon questioning, the patient admits to worsening dyspnea on exertion and generalized fatigue. He denies having fever, chills, cough, dysuria, blood loss, or weight loss. Routine laboratory studies reveal a hemoglobin concentration of 8.0 g/dl, a hematocrit of 24%, and a mean cell volume of 70 fl. The patient denies eating nonfood substances but does admit to craving and eating large amounts of ice daily. The patient’s stool is positive for occult blood by guaiac testing.

For this patient, which of the following statements regarding iron deficiency anemia is true?
A. In men and postmenopausal women, pica and a poor supply of dietary iron are the most common causes of iron deficiency anemia
B. Pagophagia, or pica with ice, is a symptom that is believed to be specific for iron deficiency
C. Measurement of the serum iron concentration is the most useful test in the detection of iron deficiency
D. The preferred method of iron replacement for this patient is parenteral therapy

Key Concept/Objective: To understand the historical components, laboratory diagnosis, and treatment of iron deficiency anemia

Blood loss is the most common cause of increased iron requirements that lead to iron deficiency. In men and postmenopausal women, iron deficiency is almost always the result of gastrointestinal blood loss. In older children, men, and postmenopausal women, a poor supply of dietary iron is almost never the only factor responsible for iron deficiency; therefore, other etiologic factors must be sought, especially blood loss. Pagophagia, or pica with ice, is thought to be a highly specific symptom of iron deficiency and disappears shortly after iron therapy is begun. Measurement of the serum ferritin concentration is the most useful test for the detection of iron deficiency, because serum ferritin concentrations decrease as body iron stores decline. A serum ferritin concentration below 12 mg/L is virtually diagnostic of absent iron stores. In contrast, a normal serum ferritin concentration does not confirm the presence of storage iron, because serum ferritin may be increased independently of body iron by infection, inflammation, liver disease, malignancy, and other conditions. Oral and parenteral replacement therapy yield similar results, but for almost all patients, oral iron therapy is the treatment of choice. Oral iron therapy is effective, safe, and inexpensive. (Answer: B—Pagophagia, or pica with ice, is a symptom that is believed to be specific for iron deficiency)

8. A 55-year-old white man with type 2 diabetes mellitus and dyslipidemia presents to your clinic for followup. He has been a patient of yours for 2 years. His diabetes has been well controlled for the past year. On review of systems, the patient states that his skin has become tan over the past several months. Routine laboratory studies show that the patient’s alanine aminotransferase level is elevated today; there are no other liver function abnormalities. Physical examination confirms that the patient’s skin is hyperpigmented with a bronze hue. You strongly suspect that this patient may have hereditary hemochromatosis.

For this patient, which of the following statements regarding HFE-associated hereditary hemochromatosis is true?
A. The classic tetrad of clinical signs associated with hemochromatosis is liver disease, diabetes mellitus, skin pigmentation, and gonadal failure
B. Measurement of the serum iron level is usually recommended as initial phenotypic screening, followed by genotypic testing
C. The treatment of choice for hemochromatosis is chelation therapy or intermittent phlebotomy for 2 or 3 months
D. In patients with hemochromatosis and cirrhosis, the rates of development of hepatocellular carcinoma equal that of the standard population

Key Concept/Objective: To know the clinical features of and appropriate therapy for HFE-associated hemochromatosis

In homozygotes who present with hereditary hemochromatosis in middle age or later, the classic tetrad of clinical signs is liver disease, diabetes mellitus, skin pigmentation, and gonadal failure. Measurement of serum transferrin saturation is usually recommended as the initial phenotypic screening determination. Although individual laboratories may have their own reference ranges, a persistent value of 45% or higher is often recommended as a threshold value for further investigation. The serum ferritin level is then used as a biochemical indicator of iron overload; in the absence of complicating factors, elevated concentrations suggest increased iron stores. Genetic testing should be considered in patients with abnormal elevations in transferrin saturation, serum ferritin, or both. The treatment of choice for hereditary hemochromatosis is phlebotomy to reduce the body iron to normal or near-normal levels and to maintain it in that range. The phlebotomy program should remove 500 ml of blood once weekly or, for heavily loaded patients, twice weekly, until the patient is iron deficient. In patients with hereditary hemochromatosis, prolonged treatment is often needed. For example, if the initial body iron burden is 25 g, complete removal of the iron burden with weekly phlebotomy may require therapy for 2 years or more. If phlebotomy therapy removes the iron load before diabetes mellitus or cirrhosis develops, the patient’s life expectancy is normal. If cirrhosis develops, however, the risk of hepatocellular carcinoma is increased by more than 200-fold. (Answer: A—The classic tetrad of clinical signs associated with hemochromatosis is liver disease, diabetes mellitus, skin pigmentation, and gonadal failure)

For more information, see Brittenham GM: 5 Hematology: II Red Blood Cell Function and Disorders of Iron Metabolism. ACP Medicine Online (www.acpmedicine.com). Dale DC, Federman DD, Eds. WebMD Inc., New York, March 2004

Anemia: Production Defects

9. A 30-year-old African-American woman presents to your office with a chief complaint of weakness. She states that she has been feeling “run down” for several weeks now. Further questioning reveals that she is diffusely weak. She is without focal deficits. She has experienced dyspnea on exertion, and she has a new rash. Physical examination is notable for mild tachycardia, pale conjunctiva, petechiae on her mucus membranes and lower extremities, and an absence of hepatomegaly or splenomegaly. Results of a complete blood cell count (CBC) are as follows: hematocrit, 21%; white blood cell count (WBC), 1,200 cells/mm3; and platelet count, 12,000 cells/mm3. The results of a bone marrow biopsy with aspirate are consistent with aplastic anemia.

Which of the following statements regarding aplastic anemia is false?
A. Aplasia can be a prodrome to hairy-cell leukemia, acute lymphoblastic leukemia, or acute myeloblastic leukemia
B. Viral infections remain one of the major causes of aplastic anemia
C. Ionizing irradiation and chemotherapeutic drugs that can be used to treat malignant or immunologic disorders can cause aplastic anemia
D. Approximately 20% of patients with hepatitis can experience aplastic anemia 2 to 3 weeks after they experience a typical case of acute hepatitis

Key Concept/Objective: To understand the etiologies of aplastic anemia

Pancytopenia (i.e., anemia, neutropenia, and thrombocytopenia) and a finding of aplastic marrow on biopsy establish a working diagnosis of aplastic anemia. Aplastic anemia has a number of causes, although in many cases the exact cause cannot be determined. Ionizing irradiation and chemotherapeutic drugs used in the management of malignant and immunologic disorders have the capacity to destroy hematopoietic stem cells. With careful dosing and scheduling, recovery is expected. Certain drugs, such as chloramphenicol, produce marrow aplasia that is not dose dependent. Gold therapy and the inhalation of organic solvent vapors (e.g., benzene or glue) can also cause fatal marrow failure. In 2% to
10% of hepatitis patients, severe aplasia occurs 2 to 3 months after a seemingly typical case of acute disease. Often, the hepatitis has no obvious cause, and tests for hepatitis A, B, and C are negative. Aplasia can also be part of a prodrome to hairy-cell leukemia, acute lymphoblastic leukemia, or, in rare cases, acute myeloid leukemia, or it can develop in the course of myelodysplasia. Parvovirus infection is the cause of the transient aplastic crises that occur in patients who have severe hemolytic disorders. The marrow in patients with such disorders must compensate for the peripheral hemolysis by increasing its production up to sevenfold. Although parvovirus can affect all precursor cells, the red cell precursors are the most profoundly affected. Anemia causes fatigue and shortness of breath; thrombocytopenia causes petechiae, oral blood blisters, gingival bleeding, and hematuria, depending on the level of the platelet count. By far the major problem is the recurrent bacterial infections caused by the profound neutropenia. The diagnosis of aplastic anemia requires a marrow aspirate and biopsy, as well as a thorough history of drug exposures, infections, and especially symptoms suggesting viral illnesses and serologic test results for hepatitis, infectious mononucleosis, HIV, and parvovirus. Measurement of red cell CD59 is helpful in the diagnosis of paroxysmal nocturnal hemoglobinuria. (Answer: D— Approximately 20% of patients with hepatitis can experience aplastic anemia 2 to 3 weeks after they experience a typical case of acute hepatitis)

10. A 43-year-old white man presents to your clinic complaining of fatigue and paresthesias. He is a vegetarian and does not take a multivitamin. His examination reveals pallor, an absence of hepatosplenomegaly, normal muscle strength throughout, and loss of position sensation and vibratory sensation distally. A CBC reveals anemia, with a mean corpuscular volume (MCV) of 106 fl. His WBC, platelet count, and serum chemistries are normal. He has had no toxic exposures and is taking no medications.

Which of the following statements about megaloblastic anemia is false?
A. Absorption of cobalamin in the small intestine is dependent on proteins produced in the mouth and stomach
B. Megaloblastic erythropoiesis is characterized by defective DNA synthesis and arrest at the G2 phase, with impaired maturation and a
buildup of cells that do not synthesize DNA and that contain anomalous DNA
C. In most patients with severe cobalamin deficiency, the neurologic examination is normal
D. Cobalamin deficiency is treated with parenteral cobalamin therapy

Key Concept/Objective: To understand the etiology, diagnosis, and treatment of pernicious anemia

Megaloblastic erythropoiesis is characterized by defective DNA synthesis and arrest at the G2 phase, with impaired maturation and a buildup of cells that do not synthesize DNA and that contain anomalous DNA. This condition leads to asynchronous maturation between the nucleus and cytoplasm. RNA production and protein synthesis continue; thus, larger cells, or megaloblasts, are produced. In addition to macrocytic and megaloblastic anemia, the patient with cobalamin deficiency may have weakness, lethargy, or dementia, as well as atrophy of the lingual papillae and glossitis. Neuropathy is the presenting feature in about 12% of patients with cobalamin (vitamin B12) deficiency without concomitant anemia. Patients with severe cobalamin deficiency initially complain of paresthesia. The sense of touch and temperature sensitivity may be minimally impaired. Memory impairment and depression may be prominent. The disease may progress, involving the dorsal columns, causing ataxia and weakness. The physical examination reveals a broadbased gait, the Romberg sign, slowed reflexes, and a loss of sense of position and feeling of vibration (especially when tested with a 256 Hz tuning fork). (Answer: C—In most patients with severe cobalamin deficiency, the neurologic examination is normal)

11. A 53-year-old woman is referred to your office by her gynecologist for management of anemia. She experienced menopause at 48 years of age and has had no further vaginal bleeding. Two years ago, her hematocrit was 36%. During her last office visit, her hematocrit was 29%, and her MCV was 107 fl. The patient is dependent on alcohol. She admits to drinking a pint of wine daily, and she engages in occasional binge drinking on weekends. She denies any other sources of blood loss or icterus. She is apparently only mildly symptomatic with some fatigue.

Which of the following statements regarding megaloblastic anemia caused by folic acid deficiency is false?
A. Serum folic acid levels more accurately reflect tissue stores than do red blood cell folic acid levels
B. Folic acid deficiency can be differentiated from cobalamin deficiency by measuring methylmalonic acid and homocysteine levels; both are elevated in cobalamin deficiency, but only homocysteine is elevated in folic acid deficiency
C. Megaloblastic anemia caused by folic acid deficiency can be masked by concurrent iron deficiency anemia, but hypersegmented polymorphonuclear cells (PMNs) should still be present on the peripheral smear
D. Folinic acid can be used to treat patients with megaloblastosis and bone marrow suppression associated with the use of methotrexate

Key Concept/Objective: To understand the diagnosis and treatment of megaloblastic anemia caused by folic acid deficiency

Patients with megaloblastic anemia who do not have glossitis, a family history of pernicious anemia, or the neurologic features described for cobalamin deficiency may have folic acid deficiency. Tests to determine folic acid deficiency vary in their accuracy. Serum folic acid levels are less reliable than red blood cell folic acid levels. A serum folic acid level of less than 2 ng/ml is consistent with folic acid deficiency, as is a red blood cell folic acid level of less than 150 ng/ml. Because the combination of folic acid and iron deficiency is common, full expression of megaloblastosis is often blocked, and the patient will have a dimorphic anemia rather than the easily identifiable macro-ovalocytosis. Hypersegmentation of PMNs persists. The serum folic acid level decreases within 2 weeks after dietary folic acid ingestion completely ceases. Therefore, many hospitalized patients have low serum folic acid levels without real tissue folic acid deprivation. In evaluating patients for folic acid deficiency, values for the levels of serum folic acid, serum cobalamin, and red blood cell folic acid must be obtained. The red blood cell folic acid level reflects tissue stores. When it is difficult but necessary to distinguish the megaloblastosis of cobalamin deficiency from that of folic acid deficiency, measurements of the serum methylmalonic acid and homocysteine levels are helpful. (Answer: A—Serum folic acid levels more accurately reflect tissue stores than do red blood cell folic acid levels)

12. A 47-year-old man with a 10-year history of type 2 diabetes presents for a routine physical examination.
His diabetes is poorly controlled, and there is evidence of retinopathy and neuropathy. He is currently receiving maximum doses of oral glipizide and metformin. His ROS is negative for cardiorespiratory symptoms. His examination reveals a blood pressure of 148/92 mm Hg and retinal changes consistent with diabetic background retinopathy. He also has decreased sensation in his feet, as evidenced by his results on monofilament neuropathy testing. Laboratory studies reveal a hemoglobin A1C level of 10.6%, a microalbumin excretion rate of 300 µg/min, and a serum creatinine level of 1.4 mg/dl. His CBC reveals normal levels of leukocytes and platelets and a hemoglobin level of 9.1 g/dl, with a mean cell volume of
85. Three follow-up examinations for the presence of fecal occult blood were negative. On repeat testing, the hemoglobin level is 9.4 g/dl; the serum iron level is 35 µg/dl; total iron-binding capacity is 230; and the ferritin level is 170 ng/L.

Which of the following is the most likely cause of this patient’s anemia

A. Bone marrow suppression
B. Sequestration of iron in the reticuloendothelial system
C. Lack of erythropoietin
D. Inhibition of folate metabolism
E. A hemolytic process

Key Concept/Objective: To understand the mechanism by which chronic disease can cause anemia

Anemia of chronic disease is generally associated with conditions that release cytokines (tumor necrosis factor–a, interleukin-1, and other inflammatory cytokines). These cytokines decrease erythropoietin production, decrease the levels of iron released from the reticuloendothelial system, and increase serum ferritin levels. Administration of erythropoietin in pharmacologic doses corrects the anemia of chronic disease by compensating for the decreased production of erythropoietin. (Answer: B—Sequestration of iron in the reticuloendothelial system)

13. A 75-year-old man is brought to the clinic for evaluation of forgetfulness that has developed slowly over the past 6 months. His wife indicates that he has occasionally become lost while out shopping and several times a day demonstrates forgetfulness in the course of attempting to complete a task. He has a history of controlled hypertension and is receiving lisinopril, 10 mg daily. He denies using alcohol, and review of systems is otherwise negative. His examination reveals a well-groomed man who appears to be as old as his stated age. His vital signs are normal, as is the rest of his examination. His Folstein Mini-Mental State Examination score is 24/30, with deficits noted in short-term memory. Thyroid-stimulating hormone test results and serum chemistries are normal. His CBC reveals a hemoglobin of 11.8 g/dl, with normal leukocyte and platelet counts. The differential reveals several granulocytes with 5 nuclear segments.

What should be the next step in the treatment of this patient?
A. Order T4 and T3 resin uptake tests
B. Start donepezil, 10 mg daily
C. Refer for a neuropsychiatric evaluation
D. Obtain an MRI of the brain
E. Determine the vitamin B12 serum level

Key Concept/Objective: To understand that neuropsychiatric symptoms may occur in patients with vitamin B12 deficiency before many of the typical hematologic changes of vitamin B12 deficiency occur

Memory impairment in the absence of hematologic changes can be a presenting symptom of vitamin B12 deficiency. Studies have shown that in 20% to 30% of patients who are deficient in vitamin B12, anemia and macrocytosis are absent. A diagnosis of vitamin B12 deficiency should be entertained in patients with neurologic disturbances in the absence of anemia and macrocytosis. The indications for vitamin B12 deficiency have broadened to include nonspecific forms of cerebral dysfunction. The finding of hypersegmented neutrophils remains a sensitive indicator of vitamin B12 deficiency. (Answer: E—Determine the vitamin B12 serum level)

14. A 27-year-old African-American man presents to the hospital with increased fatigue, which he has been experiencing for the past several months. He has been treated for sickle cell anemia in the past and has received many blood transfusions. He has been hospitalized multiple times in the past for apparent crises. He currently has no fever, and his general ROS is negative. On physical examination, his vital signs are normal. His cardiothoracic examination is normal, as is his extremity examination. His abdominal examination reveals diffuse tenderness without guarding or organomegaly. Laboratory studies reveal a hemoglobin level of 8.0 g/dl, with a mean cell volume of 90 and a reticulocyte count of 0.8%. Hemoglobin electrophoresis reveals 96% hemoglobin A and 4% hemoglobin A2. Serum chemistries reveal an LDH level of 600.

What should be the next step in the treatment of this patient

A. CT scan of the abdomen
B. Bone marrow aspiration and biopsy
C. Direct Coombs test
D. Serum folate level test
E. Administration of 2 units of packed RBCs

Key Concept/Objective: To understand the importance of a bone marrow examination in evaluating anemia of unclear etiology

A bone marrow examination is necessary to identify several of the different types of anemia associated with impaired production of RBCs. This patient presents with a history of sickle cell disease, but his current clinical symptoms are not consistent with either a hemolytic crisis (low reticulocyte count) or a pain crisis. An aplastic crisis should be entertained but is not likely, given his normal WBC and platelet counts. Although parvovirus B19 infection should be considered because it can produce pure red cell aplasia in patients with sickle cell anemia, the normal results on hemoglobin electrophoresis rule out sickle cell disease in this patient. Therefore, the specific diagnosis of his anemia is in doubt. (Answer: B—Bone marrow aspiration and biopsy)

15. A previously healthy 66-year-old man presents with a 1-month history of increasing fatigue and easy bruising. He is on no medications and consumes no alcohol. He has been retired for the past 2 years from an accounting job. His examination reveals mild pallor of the palpebral conjunctiva, brisk carotid upstrokes, and a grade 2/6 systolic ejection murmur at the left sternal border, without radiation. His abdominal examination is normal. Examination of his extremities reveals many ecchymoses in various stages of healing and a few nonpalpable petechaie on his lower extremities. A CBC reveals a WBC count of 2.4, with 20% PMNs; hemoglobin of 9.7 g/dl; a reticulocyte count of 0.5%; and a platelet count of
20,000. Bone marrow aspiration and biopsy reveal 25% normal cellularity. A diagnosis of aplastic anemia is made.

Which of the following statements is true regarding this patient’s prognosis?
A. Prognosis is relatively good, with a 70% rate of spontaneous remission within 1 year
B. Prognosis is poor, with a 70% mortality within 1 year
C. A complete cure can be expected with immunosuppressive treatment
D. Allogeneic bone marrow transplantation is associated with an 80%
5-year survival rate

Key Concept/Objective: To understand the poor prognosis for patients with severe aplastic anemia

Severe aplastic anemia is defined by a bone marrow cellularity of less than 25% normal or a cellularity of less than 50% normal with fewer than 30% hematopoeitic cells and low peripheral cell lines. The likelihood of spontaneous remission is very low, and mortality within 1 year is 70%. For mild cases, supportive treatment with blood product replacement and hematopoietic growth factor treatment is indicated; in severe cases, either immunosuppresive treatment—which would not be curative—or bone marrow transplantation may be indicated. The 5-year survival rate after bone marrow transplantation is excellent in patients younger than 49 years (86%) but drops to 54% in patients older than 60 years. (Answer: B—Prognosis is poor, with a 70% mortality within 1 year)

16. A 30-year-old woman presents for evaluation. Over the past month, she has been experiencing increasing dyspnea with exertion. She has otherwise been in good health and has no known cardiorespiratory problems. She has been consuming three or four alcoholic drinks an evening for the past 6 years. She is on no medications. In giving her family history, she reports having a sibling with anemia. Her examination reveals normal vital signs and mild pallor in the conjunctiva. Her cardiorespiratory examination is normal. A CBC reveals a WBC count of 3.5 × 103, a platelet count of 144,000, and a hemoglobin level of 8.4 g/dl, with a mean cell volume of 84 and a reticulocyte count of 0.8%.

What would be the most likely finding on this bone marrow examination?
A. Ringed sideroblasts and ineffective erythropoiesis
B. Hypercellularity of RBC precursors
C. Hypocellularity of the marrow blood-forming elements
D. Normal cellularity
E. Marrow fibrosis

Key Concept/Objective: To understand the acquired form of sideroblastic anemia

Sideroblastic anemias are a heterogeneous group of disorders characterized by anemia and the presence of ringed sideroblasts in the marrow. There are hereditary forms and acquired forms, which are further subdivided into benign and malignant variants. Abnormalities of heme synthesis are the usual causes. Iron enters the RBC precursor but cannot be incorporated and accumulates to form ringed sideroblasts. The diagnosis is established by the presence of reticulcytopenia and ringed sideroblasts in the bone marrow. Cytogenetic studies of the bone marrow may reveal changes seen in myelodysplastic syndromes. Alcohol abuse can cause a reversible form of sideroblastic anemia, and stopping alcohol is an important aspect of patient care. (Answer: A—Ringed sideroblasts and ineffective erythropoiesis)

For more information, see Schrier SL: 5 Hematology: III Anemia: Production Defects. ACP Medicine Online (www.acpmedicine.com). Dale DC, Federman DD, Eds. WebMD Inc., New York, June 2004

Hemoglobinopathies and Hemolytic Anemia

17. A 31-year-old woman presents to you for follow-up after a visit to the emergency department 1 week ago.
The patient went to the emergency department because of severe right upper quadrant pain of 2 days’ duration. An abdominal CT was normal—there was no evidence of biliary disease, nephrolithiasis, or pelvic disease. The patient was sent home that day with minimal pain control. She now states that her abdominal pain has persisted and is unimproved. She also states that she now has bloody urine in the morning and that she has developed severe lower extremity swelling and abdominal distention. Physical examination is significant for marked ascites, tender hepatomegaly, and 3+ bilateral lower extremity pitting edema. You order magnetic resonance imaging of the abdomen, which reveals the presence of a hepatic vein thrombosis.

Which of the following statements regarding paroxysmal nocturnal hemoglobinuria (PNH) is true?
A. PNH is the result of a mutation that causes a deficiency of a membrane-anchoring protein, which in turn results in an inability to properly modulate complement attack
B. PNH causes anemia but has no effect on other cell lines
C. PNH can cause thromboses in unusual sites, such as in the mesenteric or hepatic vein; however, lower extremity thromboses and associated pulmonary emboli are not seen
D. The diagnosis of PNH is a diagnosis of exclusion because there are no specific tests available for PNH

Key Concept/Objective: To know the clinical characteristics of PNH

The mutation associated with PNH results in a deficiency of the membrane-anchoring protein phosphatidylinositol glycan class A. Normal human erythrocytes, and probably platelets and neutrophils, modulate complement attack by at least three glycosylphosphatidylinositol (GPI) membrane-bound proteins: DAF (CD55), C8-binding protein (C8BP), and MIRL (CD59). Because the defective synthesis of GPI affects all hematopoietic cells, patients with PNH may have variable degrees of anemia, neutropenia, or thrombocytopenia, or they may have complete bone marrow failure. Recurrent venous occlusions lead to pulmonary embolism and hepatic and mesenteric vein thrombosis, possibly resulting from the release of procoagulant microparticles derived from platelets. Diagnosis is made by specific tests based on fluorescence-activated cell sorter analysis using antibodies that quantitatively assess DAF (CD55) and particularly MIRL (CD59) on the erythrocyte or on the leukocyte surface. (Answer: A—PNH is the result of a mutation that causes a deficiency of a membrane-anchoring protein, which in turn results in an inability to properly modulate complement attack)

18. A 48-year-old black man presents to the emergency department for evaluation of severe fatigue. He has been HIV positive for several years. He reports that his last known CD4+ T cell count was “around 100.” The patient is receiving drugs for prophylaxis against Pneumocystis carinii pneumonia (PCP). He was in his usual state of moderate health until 2 days ago. His only complaints are severe fatigue and some dyspnea on exertion. He denies having fever, chills, cough, abdominal pain, or dysuria. He states that his doctor recently changed his “PCP pill” because of a persistent rash. A chest x-ray is within normal limits. Laboratory values are remarkable for a hematocrit of 22% and a urinalysis that shows 4+ blood and
0–2 RBCs.

Which of the following statements regarding glucose-6-phosphate dehydrogenase (G6PD) deficiency is true?
A. G6PD is an enzyme that catalyzes the conversion of adenosine diphosphate (ADP) to adenosine triphosphate (ATP), a powerful reducing agent
B. G6PD deficiency is very rare in the United States
C. G6PD deficiency occurs with equal frequency in males and females
D. Potential users of dapsone should be screened for G6PD deficiency

Key Concept/Objective: To understand the function of G6PD and the epidemiology of G6PD deficiency

G6PD is the first enzyme in the pentose phosphate pathway, or hexose monophosphate shunt. It catalyzes the conversion of the oxidized form of nicotinamide-adenine dinucleotide phosphate (NADP+) to the reduced form (NADPH), which is a powerful reducing agent. NADPH is a cofactor for glutathione reductase and thus plays a role in protecting the cell against oxidative attack. G6PD deficiency is one of the most common disorders in the world; approximately 10% of male blacks in the United States are affected. The gene for G6PD is on the X chromosome at band q28; males carry only one gene for this enzyme, so those males that are affected by the disorder are hemizygous. Females are affected much less frequently because they would have to carry two defective G6PD genes to show clinical disease of the same severity as that in males. Dapsone, which is capable of inducing oxidant-type hemolysis, has increasingly come into use as prophylaxis for PCP in patients infected with HIV. Therefore, it is important to screen potential users of dapsone for G6PD deficiency with the standard enzymatic tests. (Answer: D—Potential users of dapsone should be screened for G6PD deficiency)

19. A 25-year-old black man comes for a routine office visit. You have followed the patient for many years for his sickle cell disease. The patient takes very good care of himself and has only required hospital admission four times in the past 5 years. Two of these admissions occurred in the past 6 months. You feel that the patient’s clinical course is worsening. He has recently required the addition of narcotics to his home regimen of nonsteroidal anti-inflammatory drug therapy. The patient states that he now has moderately severe pain in long bones two to three times monthly. He has also developed worsening left hip pain over the past month.

Which of the following statements regarding sickle cell disease is true?
A. In patients with homozygous sickle cell disease, roughly 50% of total hemoglobin is hemoglobin S
B. Risk factors that predispose to painful crises include a hemoglobin level greater than 8.5 g/dl, pregnancy, cold weather, and a high reticulocyte count
C. The most definitive test for the diagnosis of sickle cell anemia is the sodium metabisulfite test
D. Hydroxyurea has never been shown to be of benefit in the therapy of sickle cell disease

Key Concept/Objective: To know the clinical features of sickle cell disease

Sickle cell disease develops in persons who are homozygous for the sickle gene (HbSS), in whom 70% to 98% of hemoglobin is of the S type. About 0.2% of African Americans have sickle cell anemia. Risk factors that predispose to painful crises include a hemoglobin level greater than 8.5 g/dl, pregnancy, cold weather, and a high reticulocyte count. Conversely, the low hematocrit in sickle cell anemia reduces blood viscosity and is protective. The most definitive tests for sickle cell anemia are hemoglobin electrophoresis or high-performance liquid chromatography, which indicate the relative percentages of HbS and HbF. Hydroxyurea produces an increase in F reticulocyte and HbF levels. In a phase III trial, patients treated with hydroxyurea (starting dosage, 15 mg/kg/day) had fewer painful crises, fewer admissions for crisis, and fewer episodes of acute chest syndrome and required fewer transfusions than patients given a placebo. There was no effect on stroke; however, after 8 years of follow-up, mortality was reduced by 40%. (Answer: B—Risk factors that predispose to painful crises include a hemoglobin level greater than 8.5 g/dl, pregnancy, cold weather, and a high reticulocyte count)

20. A 47-year-old woman presents to your office with a complaint of severe fatigue, weakness, and dyspnea on exertion. She has had these symptoms for 2 days. The patient denies having fever, chills, weight changes, or dysuria. Her medical history is significant for pernicious anemia and hypothyroidism. Results of thyroid studies were within normal limits 1 week ago. Her physical examination is positive for mild icterus and hepatosplenomegaly. CBC is normal, with the exception of a hematocrit of 21%. Her hematocrit was 36% 3 months ago. Liver function tests show the total bilirubin level to be 4.6 mg/dl and the indirect bilirubin level to be 4.2 mg/dl. Other results are within normal limits. A direct Coombs test is positive.

Which of the following statements regarding autoimmune hemolytic anemia is true?
A. Autoimmune hemolytic anemia typically results in intravascular hemolysis
B. Autoimmune hemolytic anemia may be idiopathic or secondary to disorders such as systemic lupus erythematosus, chronic lymphocytic leukemia (CLL), HIV infection, or hepatitis C infection
C. Most patients with autoimmune hemolytic anemia are cured with steroid therapy
D. Splenectomy is curative for those patients who do not respond to simple steroid therapy

Key Concept/Objective: To understand the clinical features and chronic nature of autoimmune hemolytic anemia

Intravascular hemolysis in autoimmune hemolytic anemia is rare and indicates that an extremely rapid rate of erythrocyte destruction is occurring or that the extravascular removal mechanisms have been overwhelmed. Erythrocytes sensitized to IgG alone are usually removed in the spleen, whereas RBCs sensitized to IgG plus complement or to complement alone are generally destroyed in the liver, because hepatic Kupffer cells carry receptors specific for complement component C3b. Both an idiopathic variety of autoimmune hemolytic anemia and a variety that occurs secondary to other disorders have been described. Such primary disorders include systemic lupus erythematosus, non-Hodgkin lymphoma (especially chronic lymphocytic leukemia), Hodgkin disease, cancer, myeloma, dermoid cyst, HIV infection, angioimmunoblastic lymphadenopathy with dysproteinemia, hepatitis C infection, and chronic ulcerative colitis. Approximately 20% of patients remain well indefinitely after steroid therapy, but the majority suffer from a chronic, treacherous disease that can produce sudden relapses with abrupt anemia. If the corticosteroid dose required for long-term therapy produces significant morbidity, one can proceed empirically either to splenectomy or to the use of immunosuppressive agents. Splenectomy rarely results in extended remission but is valuable as a prednisone-sparing measure. (Answer: B—Autoimmune hemolytic anemia may be idiopathic or secondary to disorders such as systemic lupus erythematosus, chronic lymphocytic leukemia [CLL], HIV infection, or hepatitis C infection)

21. A 19-year-old African-American man comes to establish primary care. He knows that both of his parents are carriers of sickle cell anemia. He also knows that one of the parents has a second hemoglobin defect. He has never experienced pain crises.

Which of the following compound heterozygote states, if present, would make the patient susceptible to symptoms such as pain crises and avascular necrosis?
A. HbS/HbC
B. HbS/HbE
C. HbS/a-thalassemia
D. HbS/a+-thalassemia
E. HbS/a0-thalassemia

Key Concept/Objective: To understand the mechanisms of sickling and the protective role of globin chains

Certain physiologic stresses make deoxygenated HbS more likely to polymerize and precipitate, forming a gel-like substance. Known stressors are a low pH, an increase in the level of intracorpuscular hemoglobin (MCHC), a high concentration of HbS, a low concentration of HbF, and slow transit time. Patients with a+-thalassemia traits produce enough aAglobin chains to maintain an HbA concentration greater than 25%, thus decreasing the amount of HbS. Patients with a-thalassemia tend to have lower levels of MCHC and, thus, decreased concentrations of HbS. HbE is an unstable hemoglobin in which lysine is substituted for glutamic acid in position 26 of the a-globin chain. Heterozygous patients are asymptomatic. Homozygous patients present with a clinical picture similar to that of patients with a+-thalassemia. Similarly, patients with HbS/HbE have a clinical picture similar to that of patients with HbS/a+-thalassemia. In patients with HbS/HbC, 50% of the hemoglobin is HbS. Although HbS/HbC disease is less severe than HbSS disease, patients with HbS/HbC frequently tend to have avascular necrosis of the femoral head. Patients with a0-thalassemia have a very low concentration of HbA; in these patients, HbS constitutes more than 50% of the intracorpuscular hemoglobin. In extreme cases, the clinical picture of a0-thalassemia is identical to that of HbSS. As HbS decreases, the clinical picture comes to resemble that of patients with HbS/HbC disease. (Answer: E—HbS/a0-thalassemia)

22. A 38-year-old African-American man is admitted to the hospital with congestive heart failure (CHF) of new onset. He is noted to have a blood pressure of 210/140 mm Hg. Therapy with intravenous furosemide, intravenous nitroglycerin, and oral angiotensin-converting enzymes controls his symptoms and blood pressure over the next 48 hours. On the third day, a 10% drop of his hematocrit is noted. Laboratory data show the following: Hb, 11 g/dl (admission Hb, 14.5 g/dl); Hct, 35% (admission Hct,
45%); total bilirubin, 3.5 mg/dl (indirect, 2.8 mg/dl); LDH, 550 mg/dl; haptoglobin level, undetectable.

Which of the following tests is most likely to establish the diagnosis?
A. A test for immune hemolysis (i.e., direct Coombs test, indirect Coombs test, or both)
B. A test for membrane fragility (i.e., sucrose lysis test)
C. A test for red cell enzyme deficiency (e.g., pyruvate kinase assay)
D. Hemoglobin electrophoresis

Key Concept/Objective: To understand hemolysis secondary to use of oxidating agents (furosemide and nitroglycerin) and the timing of the G6PD assay

This patient experienced an episode of acute hemolysis after being hospitalized. It is likely that this is a case of drug-induced hemolysis. There are several mechanisms by which drugs can induce hemolysis; two well-recognized mechanisms are immunologic mediation (e.g., hemolysis caused by penicillin and methyldopa) and an increase in oxidative stress on red cells. Oxidative stress can occur as a result of hemoglobins becoming unstable or through a decrease in reduction capacity (as would result from G6PD deficiency). Penicillins and cephalosporins produce immune hemolysis by acting as a hapten in the red cell membrane. The protein/drug complex elicits an immune response. An IgG antibody is generated that acts against the drug-red cell complex. In such patients, the direct Coombs test is positive, but the indirect Coombs test is negative. Other drugs induce hemolysis by altering a membrane antigen. IgG autoantibodies that cross-react with the native antigen are produced. The direct Coombs test is also positive in this form of drug reaction. Methyldopa is the classic example of this form of interaction, although other drugs such as procainamide and diclofenac have been clearly implicated. Diclofenac can produce massive hemolysis with concomitant disseminated intravascular coagulation and shock. Sucrose lysis is still used to screen for membrane fragility. The most common disorder associated with this abnormality is paroxysmal nocturnal hemoglobinuria (PNH). The lack of associated cytopenias, the acuteness of the onset of symptoms, and the lack of history of venous thrombosis (especially thrombosis at unusual sites such as the inferior vena cava or the portal mesenteric system or thrombosis that produces Budd-Chiari syndrome) makes PNH an unlikely cause of this patient’s symptoms. Some unstable hemoglobins, such as HbE, are susceptible to hemolysis from oxidative stress. This patient was exposed to both furosemide (a drug with a sulfa moiety) and nitroglycerin. This hemoglobinopathy is diagnosed by hemoglobin electrophoresis. However, this disease is seen almost exclusively in individuals from Southeast Asia (Cambodia, Thailand, and Vietnam). The most likely diagnosis in this case is G6PD deficiency. This enzymopathy affects 10% of the world population. It is commonly seen in African Americans. The red cell becomes hemolyzed when exposed to an oxidative stress. Older red cells are more susceptible to hemolysis because levels of G6PD decrease as red cells age. The results of the G6PD assay should be interpreted carefully. On occasion, the results of the G6PD assay will be normal in patients with G6PD deficiency; this occurs when the assay detects G6PD in very young cells (reticulocytes) that are being released as a result of the brisk hemolysis. (Answer: D— Hemoglobin electrophoresis)

23. A 17-year-old African-American woman is referred to you from the blood bank for evaluation of microcytic anemia detected at the time of screening for blood donation. She is unaware of this problem. She has no symptoms of fatigue or dyspnea. Her menstrual period appears to be normal in frequency and volume of blood loss. There is no history of GI bleeding. She has not been pregnant before. Her physical examination is unremarkable. Her laboratory values are as follows: Hb, 10.5 g/dl; Hct, 32%; MCV, 61; WBC, 7,500 with normal differential; platelet count, 235,000; peripheral smear shows hypochromia and microcytosis; red cell count, 5 million cells/mm3; Hb electrophoresis: HbA, 97%; HbA2, 2%; HbF, < 1%.

What is the most likely diagnosis for this patient?
A. Iron deficiency anemia
B. a-Thalassemia minor
C. Heterozygous a-thalassemia-1
D. Hemoglobin H disease
E. Homozygous a-thalassemia-2

Key Concept/Objective: To understand the interpretation of the red cell count in patients with anemia, the results of hemoglobin electrophoresis in patients with thalassemia, and differences in genotype among the thalassemias

Microcytic and hypochromic anemia is common in clinical practice. The most likely cause is iron deficiency anemia, especially in women of childbearing age. In iron deficiency anemia, the production of red cells is deficient. Thus, the red cell count tends to be low when anemia is present. This patient has a normal red cell count and mild to moderate anemia. This combination should suggest a problem in red cell production. Both a and ß-thalassemia can present as an asymptomatic microcytic/hypochromic anemia. In ß-thalassemia, there is a decrease in production of ß-globin chains. The a-globin chains are produced in normal amounts and combine with other globin chains. As such, HbA2 production is increased. This change in hemoglobin patterns enables the diagnosis of ß-thalassemia to be made through use of hemoglobin electrophoresis. With a-thalassemias, the hemoglobin electrophoresis pattern is normal. This globin chain is under control of two different loci with a total of four alleles. The absence of both the a-globin gene and the ßglobin gene causes hemoglobin Barts syndrome during fetal life. This hemoglobinopathy results in children being born with hydrops fetalis. When three alleles are missing, the patient develops HbH disease. This disease is seen exclusively in patients of Mediterranean origin and is characterized clinically by chronic hemolysis and the presence of splenomegaly. All three features are absent in this patient. Of the two remaining possibilities, we can conclude that the patient has a-thalassemia-2 because of the ethnic origin. This disease is characterized by homozygous inheritance (–a/-a) and is seen exclusively in African Americans. The heterozygous patient of this genotype (–a/a a) is the silent carrier. On the other hand, patients with a-thalassemia-1 have lost two a-chain genes in one chromosome (—/a a). This genotype is seen almost exclusively in Asians. (Answer: E— Homozygous a-thalassemia-2)

For more information, see Schrier SL: 5 Hematology: IV Hemoglobinopathies and Hemolytic Anemias. ACP Medicine Online (www.acpmedicine.com). Dale DC, Federman DD, Eds. WebMD Inc., New York, April 2004

The Polycythemias

24. A 60-year-old man presents with complaints of headache, light-headedness, blurry vision, and fatigue; these symptoms have been increasing over the past month. He reports that he has felt weak and has not had much energy. He also reports generalized itching, which usually occurs after he takes a hot shower. Physical examination reveals facial plethora. His spleen is palpable 2 cm below the left costophrenic angle. Laboratory results reveal the following: Hb, 18; Hct, 61; platelets, 500,000; leukocytes, 17,000.

Which of the following is the most appropriate diagnosis for this patient?

A. Gaisböck syndrome (relative polycythemia)
B. Pickwickian syndrome
C. Polycythemia vera
D. Acute myeloid leukemia
E. Chronic myeloid leukemia

Key Concept/Objective: To understand the clinical characteristics of polycythemia vera and primary polycythemia and differentiate these characteristics from those of other types of polycythemia

Polycythemia vera is an acquired myeloproliferative disorder that is characterized by overproduction of all three hematopoietic cell lines with predominant elevation in red cell counts. This overproduction is independent of erythropoietin. Facial plethora is characteristic of all patients with polycythemia vera. Polycythemia vera is slightly more common in men than in women and is most frequently diagnosed in persons between 60 and 75 years of age. Patients commonly complain of pruritus, especially after a hot bath. Splenomegaly is also common. Gaisböck syndrome, also known as relative polycythemia, is usually found in men from 45 to 55 years of age; they are most often obese, hypertensive men who may also be heavy smokers. Pickwickian syndrome, or obesity-hypoventilation syndrome, is characterized by obesity with hypoxemia and hypercapnia; some patients experience nocturnal obstructive sleep apnea and daytime hypersomnolence. Although polycythemia vera and chronic myeloid leukemia are both classified as myeloproliferative disorders, the dominant features of chronic myeloid leukemia are dramatic leukocytosis, the presence of the Philadelphia chromosome, and certain evolution to acute myeloid leukemia. (Answer: C—Polycythemia vera)

25. A 62-year-old woman presents with a history of intermittent headache and vertigo. She finds it hard to concentrate. On questioning, she also complains of tinnitus. She has lost 10 lb over the past 6 months and frequently feels tired. She also complains of excessive sweating. On physical examination, she has facial plethora, her temperature is normal, her blood pressure is 130/80 mm Hg, and her heart rate is 90 beats/min. There is a soft systolic murmur at the lower sternal border, the lungs are clear on auscultation, and the spleen can be palpated 6 cm below the left costal margin. Laboratory evaluation reveals the following: Hct, 60%; WBC, 15,000/mm3; platelets, 400,000/mm3; oxygen saturation, 98%.

What is the most appropriate initial treatment for this patient?
A. Allogeneic stem cell transplantation
B. Phlebotomy
C. Hydroxyurea
D. Anagrelide

Key Concept/Objective: To recognize phlebotomy as a frequently used treatment for polycythemia

This patient has signs and symptoms characteristic of polycythemia vera; initial treatment might include phlebotomy, although no consensus has emerged about the best treatment approach. If phlebotomy is chosen as a treatment option, it should be continued at the rate of once or twice a week until the target hematocrit value of less than 45% is achieved. The patient must be instructed not to take multivitamins that contain iron. Modest iron deficiency is a desirable consequence of phlebotomy, as it helps maintain the hematocrit in the desired range. Most patients who are initially treated with phlebotomy alone will eventually require myelosuppressive therapy. Hydroxyurea is generally used for patients older than 70 years or for those who have previously had a thrombotic event or who require high-maintenance phlebotomy. Anagrelide is very expensive and has a number of side effects; it is useful in patients with polycythemia vera who require a supplemental agent for optimal control of thrombocytosis. Allogeneic stem cell transplantation is a highly investigational approach; only a few patients with polycythemia vera have undergone myeloablative therapy and allogeneic bone marrow transplantion. (Answer: B—Phlebotomy)

26. A 75-year-old man presents complaining of headache and memory difficulties. He has lost 10 lb since his last office visit 6 months ago. He smoked cigarettes for 40 years but was successful in quitting 10 years ago. He has a history of deep vein thrombosis in his left leg, which was successfully treated by medication. His blood pressure is 135/85 mm Hg, and his temperature is 101° F (38.3° C). He complains of feeling flushed and of sweating. His laboratory values are as follows: Hct, 60%; WBC, 15,000/mm3; platelets,
400,000/mm3; serum erythropoietin, 0.5 U/L.

What is the most appropriate treatment for this patient?
A. Phlebotomy
B. Phlebotomy and hydroxyurea
C. Hydroxyurea
D. Anagrelide

Key Concept/Objective: To understand that patients with polycythemia vera who are older than 70 years should be treated with a myelosuppressive agent such as hydroxyurea in combination with phlebotomy

The elevated hematocrit level, reduced serum erythropoietin level, headache, fever, memory difficulties, and weight loss seen in this patient are characteristic of polycythemia vera. Although there is not an overall consensus on treatment of polycythemia vera, phlebotomy is frequently used as an initial treatment. Because this patient is older than 70 years and has previously had a thrombotic event, a myelosuppressive agent such as hydroxyurea should be used in addition to phlebotomy. This combination of hydroxyurea and phlebotomy has been demonstrated to be an effective therapeutic regimen in controlling the hematocrit in most patients with polycythemia vera and in lowering the risk of thrombosis that occurs with use of phlebotomy alone. When hydroxyurea is used, complete blood counts should be frequently monitored to avoid excessive myelosuppression. Patients may experience painful leg ulcers. Two reports have shown an increased risk of acute myeloid leukemia in patients taking hydroxyurea, although it remains controversial whether the use of hydroxyurea increases the risk of acute myeloid leukemia. (Answer: B—Phlebotomy and hydroxyurea)

27. A 45-year-old man presents with weakness and shortness of breath. He complains of a headache, fatigue, light-headedness, and ringing in his ears. He smokes three packs of cigarettes a day and has smoked for 30 years. He is 5 ft 8 in tall and weighs 280 lb. On physical examination, his complexion is ruddy. His blood pressure is 140/85 mm Hg. There is a history of heart disease in his family. He has been treated for hypertension for the past 10 years with a therapeutic regimen consisting of an antihypertensive agent and a diuretic. Laboratory reports reveal the following: Hct, 57%; red cell mass, 34 ml/kg; low-normal plasma volume; oxygen saturation, 97%.

Which of the following is the most likely diagnosis for this patient?

A. Gaisböck syndrome (relative polycythemia)
B. Pickwickian syndrome
C. Polycythemia vera
D. Acute myeloid leukemia
E. Chronic myeloid leukemia

Key Concept/Objective: To recognize that middle-aged, obese, hypertensive men who are heavy smokers and who are being treated with diuretics may have Gaisböck syndrome even if their hematocrit levels are lower than 60%

The red cell mass of less than 36 ml/kg, reduced oxygen levels, and low-normal plasma volume seen in this patient suggest a diagnosis of Gaisböck syndrome. Gaisböck syndrome, or relative polycythemia, is often seen at an earlier age (45 to 55 years) than polycythemia vera. In the male population in the United States, 5% to 7% have Gaisböck syndrome. Those affected are usually middle-aged, obese, hypertensive men who may also be heavy smokers. Smoking-induced elevations in the level of carboxyhemoglobin or hypoxemia may play a role in the development of Gaisböck syndrome. Long-term exposure to carbon monoxide results in chronically high levels of carboxyhemoglobin. Carbon monoxide binds to hemoglobin with an affinity many times greater than oxygen, decreasing the quantity of hemoglobin available for oxygen transport. Thus, long-term carbon monoxide exposure in cigarette and cigar smokers may cause polycythemia. In this patient, diuretic use for treatment of hypertension may also have exacerbated the deficit in plasma volume. Before treatment with phlebotomy, patients may be taken off diuretics and encouraged to lose weight and stop smoking. (Answer: A—Gaisböck syndrome [relative polycythemia])

For more information, see Broudy VC: 5 Hematology: V The Polycythemias. ACP Medicine Online (www.acpmedicine.com). Dale DC, Federman DD, Eds. WebMD Inc., New York, August 2004

Nonmalignant Disorders of Leukocytes

28. A 21-year-old man presents to the emergency department for evaluation of pain and fever. One week ago, the patient was involved in a head-on motor vehicle accident; he was not wearing a seat belt. At that time, the patient underwent an emergent resection of his spleen. The patient states that for the past
2 days, he has been experiencing swelling and redness of his incision site, as well as fever. On physical examination, the patient’s temperature is 102° F (38.9° C). Diffuse swelling and induration is noted at his incision site, and diffuse erythema surrounds the incision. Laboratory values are remarkable for a white blood cell (WBC) count of 26,000/mm3 and a differential with 50% neutrophils and 22% band forms.

Which of the following statements regarding neutrophilia is true

A. Neutrophilia is usually defined as a neutrophil count greater than
1,000/mm3
B. Thrombocytosis is commonly associated with splenectomy, but splenectomy has no association with neutrophilia
C. Serious bacterial infections are usually associated with changes in the number of circulating neutrophils, as well as the presence of younger cells, but they are not associated with changes in neutrophil morphology
D. With serious bacterial infections, characteristic morphologic changes of the neutrophils include increased numbers of band forms and increased numbers of cells with Dohle bodies and toxic granulations

Key Concept/Objective: To know the definition and morphologic characteristics of neutrophilia

Neutrophilia, or granulocytosis, is usually defined as a neutrophil count greater than 10,000/mm3. Neutrophilia most often occurs secondary to inflammation, stress, or corticosteroid therapy. Neutrophilia is also associated with splenectomy. Serious bacterial infections and chronic inflammation are usually associated with changes in both the number of circulating neutrophils and their morphology. Characteristic changes include increased numbers of young cells (bands), increased numbers of cells with residual endoplasmic reticulum (Dohle bodies), and increased numbers of cells with more prominent primary granules (toxic granulation). These changes are probably caused by the endogenous production of granulocyte colony-stimulating factor or granulocyte-macrophage colony-stimulating factor and are also seen with the administration of these growth factors. (Answer: D—With serious bacterial infections, characteristic morphologic changes of the neutrophils include increased numbers of band forms and increased numbers of cells with Dohle bodies and toxic granulations)

29. A 61-year-old woman visits your clinic for a follow-up visit. She has been coming to you for several weeks with complaints of diffuse rash, intermittent fevers, persistent cough, and dyspnea. Laboratory results were significant only for a WBC count of 15,000/mm3 with 40% eosinophils. You have completed an extensive workup for underlying allergy, connective tissue disease, malignancy, and parasitic infection, with negative results. A bone marrow biopsy revealed hypercellular marrow with eosinophils constituting 50% of the marrow elements. Your working diagnosis is hypereosinophilic syndrome (HES).

Which of the following statements regarding HES and eosinophilia is true?
A. The criteria used to diagnose HES are an unexplained eosinophil count of greater than 1,500/mm3 for longer than 6 months and signs or symptoms of infiltration of eosinophils into tissues
B. The term HES is often used for patients with chronic eosinophilia resulting from parasitic infection
C. Eosinophilia is defined as an eosinophil count greater than 500/mm3
D. Long-term corticosteroid therapy is the only available therapy for HES

Key Concept/Objective: To understand HES

Evaluation of the patient with eosinophilia (i.e., a patient with an eosinophil count greater than 700/mm3) is difficult because the causes of this disorder are multiple and diverse. The term HES is often used for patients with chronic eosinophilia of unknown cause. The criteria used to diagnose HES are an unexplained eosinophil count of greater than 1,500/mm3 for longer than 6 months and signs or symptoms of infiltration of eosinophils into tissues. If symptoms involving the lungs or the heart are present, prednisone at a dosage of 1 mg/kg/day should be given for 2 weeks, followed by 1 mg/kg every other day for 3 months or longer. If this treatment fails or if an alternative is necessary to avoid steroid side effects, hydroxyurea at a dosage of 0.5 to 1.5 g/day should be given to lower the WBC count to less than 10,000/mm3 and the eosinophil count to less than 5,000/mm3. Studies suggest that treatment with imatinib mesylate is effective. Alternative agents include interferon alfa, cyclosporine, and etoposide. (Answer: A—The criteria used to diagnose HES are an unexplained eosinophil count of greater than 1,500/mm3 for longer than 6 months and signs or symptoms of infiltration of eosinophils into tissues)

30. A 52-year-old man presents to your office for his yearly physical examination. He is completely asymptomatic and is tolerating his single blood pressure medication well. On review of systems, the patient states that for several weeks, he has had severe pruritus after showering and that lately his face has been feeling “flushed.” He has no history of food allergy or other allergy. Physical examination is significant only for facial plethora and moderate nontender splenomegaly. CBC reveals a WBC of 13,000/mm3 with
6% basophils, a hematocrit of 60%, and a platelet count of 640,000/mm3. Hemoglobin oxygen saturation on room air is 98% with a normal respiratory rate. The serum erythropoietin level is low.

Which of the following statements regarding basophils or basophilia is true?
A. Thrombocytosis is commonly associated with splenectomy, but splenectomy has no association with basophilia
B. Basophilia is defined as a basophil count greater than 500/mm3
C. Basophilia is seen in myeloproliferative disorders (MPDs) such as chronic myelogenous leukemia (CML), polycythemia vera (PV), and myeloid metaplasia, as well as in some hemolytic anemias and Hodgkin disease
D. Granules in the basophil do not contain histamine

Key Concept/Objective: To understand basophil physiology and the clinical significance of basophilia

Basophilia (i.e., a basophil count of greater than 150/mm3) is seen in MPDs such as CML, PV, and myeloid metaplasia; after splenectomy; in some hemolytic anemias; and in Hodgkin disease. The basophil count can also be increased in patients with ulcerative colitis or varicella infection. Although basophils and mast cells are involved in immediate hypersensitivity reactions and basophils are often seen in areas of contact dermatitis, basophilia is not seen in patients with these disorders. Most, if not all, of the circulating histamine in the body is synthesized by the basophil and stored in its granules. Degranulation causes the release of histamine, which mediates many immediate hypersensitivity effects and which, because it is a potent eosinophil chemoattractant, draws eosinophils to the site of degranulation. Other substances that are released on basophil degranulation include additional eosinophil chemotactic factors and a variety of arachidonic acid metabolites, the most important of which is leukotriene C4. (Answer: C—Basophilia is seen in myeloproliferative disorders [MPDs] such as chronic myelogenous leukemia [CML], polycythemia vera [PV], and myeloid metaplasia, as well as in some hemolytic anemias and Hodgkin disease)

31. You are asked to consult on a case involving a 26-year-old man who developed leukocytosis after a motorcycle accident 3 days ago. The patient has multiple fractures of the pelvis and lower extremities, extensive soft-tissue injury, and aspiration pneumonia. His leukocyte count was 35,000 on admission and has subsequently ranged up to 50,000. Currently, the patient is sedated and is on a ventilator. He is being treated with I.V. antibiotics. Physical examination is remarkable for right lower lateral consolidation, ecchymoses of the lower extremities, and the absence of hepatosplenomegaly. Laboratory values are as follows: hemoglobin, 9.5 g/dl; platelets, 140,000/mm3; WBC, 55,000/mm3, with 95% neutrophils and bands. No myelocytes or metamyelocytes are noted, and there is no elevation of the basophil or eosinophil count. The leukocyte alkaline phosphatase (LAP) score is 140 µm/L.

Which of the following cannot be the cause of this patient’s elevated neutrophil count?
A. Sepsis
B. Hemorrhage
C. Tissue injury
D. CML
E. Cytokine release

Key Concept/Objective: To understand the causes of the leukemoid reaction and distinguish them from malignant causes

The term leukemoid reaction is used to describe a profound leukocytosis (generally defined as a leukocyte count exceeding 25,000 to 30,000/mm3) that is not leukemic in etiology. Leukemoid reactions are the response of normal bone marrow to cytokine release by lymphocytes, macrophages, and other cells in response to infection or trauma. In a leukemoid reaction, the circulating neutrophils are usually mature and are not clonally derived. The major differential diagnosis is with regard to CML. Leukemoid reactions should also be distinguished from leukoerythroblastic reactions: the presence of immature white cells and nucleated red cells in the peripheral blood irrespective of the total leukocyte count. Although less common than leukemoid reactions in adults, leukoerythroblastosis reflects serious marrow stimulation or dysfunction and should prompt bone marrow aspiration and biopsy, unless it occurs in association with severe hemolytic anemia, sepsis in a patient with hyposplenism, or massive trauma. In such patients, trauma, hemorrhage, and infection all will contribute to a potent cytokine release and marrow stimulation. The absence of splenomegaly, leukocyte precursors (myelocytes, metamyelocytes), basophilia, or eosinophilia all point away from CML, and the elevated LAP score confirms the diagnosis of a leukemoid reaction. (Answer: D—CML)

32. On routine examination, a 45-year-old man is found to have a neutrophil count of 1,100/mm3. He feels well, takes no medications, and has no history of infection. His medical records reveal a persistent, asymptomatic neutropenia of 1,000 to 1,800 neutrophils/mm3 over the past 10 years.

Which of the following ethnicities would help explain this patient’s low leukocyte count?
A. Native American
B. Ashkenazi Jew
C. Yemenite Jew
D. Hmong
E. Inuit

Key Concept/Objective: To be able to recognize constitutional causes of neutropenia in certain populations

Neutropenia is present when the peripheral neutrophil count is less than 1,000 to 2,000 cells/mm3. The normal range in Africans, African Americans, and Yemenite Jews is lower. In these populations, neutrophil counts of 1,500/mm3 are common, and neutrophil counts as low as 100/mm3 are probably normal. Evaluation should focus on a history of unusual infections, medications, or toxic exposures. If these factors are absent and if previous asymptomatic neutropenia can be documented, no further evaluation or special precautions are needed. (Answer: C—Yemenite Jew)

33. A 59-year-old woman with severe, progressive rheumatoid arthritis is found to have a neutrophil count of 1,200/mm3 on routine hematologic testing. She takes methotrexate and prednisone for her rheumatoid arthritis. In addition to rheumatoid nodules and rheumatoid joint deformities, moderate splenomegaly is noted on physical examination.

Which of the following would not be contributing to this patient’s neutropenia?
A. Methotrexate
B. Corticosteroids
C. Antineutrophil antibodies
D. Felty syndrome
E. Large granular lymphocyte syndrome

Key Concept/Objective: To understand the various causes of neutropenia in rheumatoid arthritis

With the exception of prednisone, each of the listed factors can lead to neutropenia in patients with rheumatoid arthritis. Methotrexate (as well as gold, penicillamine, and other disease-modifying agents) can cause severe leukopenia and neutropenia and require CBC monitoring during therapy. Prednisone and other corticosteroids, however, do not lower the neutrophil count. Indeed, the neutrophil count rises acutely after corticosteroid administration, owing to demargination. Felty syndrome is the triad of rheumatoid arthritis, splenomegaly, and neutropenia and frequently includes hepatomegaly, lymphadenopathy, fever, weight loss, anemia, and thrombocytopenia. Leg ulcers and hyperpigmentation may also be seen. This syndrome develops late in the course of chronic, seropositive rheumatoid arthritis, often after the inflammatory arthritis has resolved. Recurrent infections with gram-positive organisms can be a serious clinical problem, and infections do not always correlate with the severity of neutropenia. Large granular lymphocyte syndrome is a clonal expansion of CD2+, CD3+, CD8+, CD16+, and CD57+ cells and is frequently associated with rheumatoid arthritis. Patients present with neutropenia, infections, and possibly splenomegaly and may be misdiagnosed as having Felty syndrome. Unlike patients with Felty syndrome, however, these patients present at an older age. Their neutropenia may develop within months of the onset of arthritis and is usually associated with a normal or elevated blood leukocyte (mostly lymphocytes) count. The clonal expansion may evolve into a lymphocytic leukemia. Peripheral blood smear shows increased numbers of large granular lymphocytes with abundant pale cytoplasm and prominent azurophilic granules. (Answer: B—Corticosteroids)

34. A 56-year-old man has felt unwell for 6 months. He complains of cough, exertional shortness of breath, paroxysmal nocturnal dyspnea, diarrhea, low-grade fever, and weight loss. He takes no medications and has not traveled outside the United States. On examination, his blood pressure is 110/50 mm Hg; his pulse is 96 beats/min and irregular; his respiration rate is 20; and his temperature is 99.3° F (37.4° C). Fine rales are present in the lower two thirds of the lung fields. The jugular venous pressure is estimated to be
15 cm, and a large V wave is present. A 3/6 holosystolic murmur is noted at the apex. A distinct S3 gallop is audible; 2+ pitting edema is present in the ankles. Chemistry panel is normal, but the CBC reveals a WBC of 22,000, of which 60% are eosinophils. Electrocardiogram shows atrial fibrillation. Chest x-ray shows interstitial and alveolar edema and Kerley B lines. Echocardiogram reveals mitral regurgitation and features suggesting a restrictive cardiomyopathy. Multiple stool samples are negative for ova and para-
sites.

Which of the following is the most likely diagnosis for this patient?
A. Asthma
B. Parasitic infestation
C. Eosinophilic leukemia
D. Hypersensitivity pneumonitis
E. Hypereosinophilic syndrome

Key Concept/Objective: To understand the diagnosis of hypereosinophilic syndrome

Hypereosinophilic syndrome consists of a chronic, unexplained eosinophilia without obvious cause (such causes would include parasitic infections, drug reactions, allergic reactions, hypersensitivity reactions, lymphoproliferative disorders, connective tissue disorders, and hematologic malignancies). The condition may be caused by excessive IL-5 production by a T cell clone. Eosinophils are found in the involved tissues and are thought to cause damage by the local deposition of toxic eosinophil products such as eosinophil major basic protein. Manifestations are multisystemic: fever, rash, cough and dyspnea, diarrhea, congestive heart failure, and peripheral neuropathy. The most severe complications involve the heart and CNS. Careful evaluation is necessary to exclude other causes of eosinophilia. Endomyocardial fibrosis (Löffler endocarditis) is a cardiac manifestation of hypereosinophilic syndrome. Eosinophilic deposits may lead to direct injury of the endocardium, followed by platelet thrombi and fibrosis. The cardiac apices can become obliterated, creating a characteristic finding on echocardiography. The mitral and tricuspid valves are affected by the same fibrotic process, resulting in valvular regurgitation. (Answer: E—Hypereosinophilic syndrome)

For more information, see Dale DC: 5 Hematology: VII Nonmalignant Disorders of Leukocytes. ACP Medicine Online (www.acpmedicine.com). Dale DC, Federman DD, Eds. WebMD Inc., New York, April 2004

Transfusion Therapy

35. A 57-year-old diabetic man presents to your office for presurgical evaluation for total hip replacement.
After a thorough history and physical examination, you ask the patient if he has any questions. He says that the orthopedic surgeon told him that there was a possibility he will need blood products during or after the surgery, and the patient is concerned about the risks of contracting a contagious disease from blood products.

Which of the following statements about the risk of infection associated with transfusion is true?
A. Risk of transfusion-associated hepatitis A is higher in pooled products such as factor concentrates than in single-donor products
B. Postdonation screening to identify donors likely to transmit bloodborne infections has produced the biggest decrease in the risk of transfusion-transmitted disease
C. Directed donation offers a small but significant reduction in the risk of transfusion-associated infections
D. Currently, there is no postdonation test available for West Nile virus

Key Concept/Objective: To understand the screening process used to reduce transmission of infectious diseases through transfusion of blood products

Available prevalence data show that the risk of infectious disease from directed donors is no different from that of first-time donors. Predonation donor screening to identify clinical and lifestyle characteristics associated with higher incidences of infection has produced the biggest decrease in the risk of transfusion-transmitted disease. Postdonation testing is essential in identifying donors likely to transmit blood-borne infections who are missed in the initial screening process. Because the viremic phase of hepatitis A lasts about 17 days in humans before signs and symptoms develop, hepatitis A transmission from singledonor products is extremely rare. Pooled products, such as factor concentrates, however, carry a substantially higher risk. Transmission of West Nile virus by blood products has led to new donor questions to eliminate donors at risk for this disease. A nucleic acid–based test for all donated units was introduced in June 2003. (Answer: A—Risk of transfusion-associated hepatitis A is higher in pooled products such as factor concentrates than in single-donor products)

36. A 49-year-old woman is admitted to the hospital with newly diagnosed severe anemia. Her hemoglobin level is 7 g/dl, and she has shortness of breath and fatigue. She denies any obvious source of blood loss, such as menorrhagia or rectal bleeding. On examination, the patient is pale. She is tachycardic, with a pulse of 110 beats/min. Her blood pressure is 105/62 mm Hg. Rectal examination shows heme-positive brown stool. Before you leave the room to write your orders, you explain the risks and benefits of blood product transfusion.

Which of the following statements about blood components is true?
A. Whole blood transfusion would be preferable to red cell transfusion in this patient
B. Leukocyte reduction reduces febrile transfusion reactions
C. Cryoprecipitate consists of albumin and platelets
D. Single-donor platelet transfusions carry a higher risk of blood-borne infection than platelet concentrates

Key Concept/Objective: To understand the components of whole blood

Except for some autologous blood programs that use whole blood rather than packed red cells, use of whole blood has now been almost completely supplanted by therapy employing specific blood components. To prevent transfusion reactions or to delay alloimmunization, red cells are further processed by leukocyte reduction or washing to remove plasma proteins. Current filter technology reduces white cell counts to less than 5 × 106 cells per unit, a concentration that is sufficient to reduce febrile transfusion reactions and delay alloimmunization and platelet refractoriness. With single-donor platelet therapy, there is a reduction in the risk of blood-borne infection and antigen exposure, because the product is from one donor rather than four to six; disadvantages are a longer collection time, greater cost, and often limited supply. Fresh frozen plasma (FFP) that is frozen within 8 hours of collection contains all the procoagulants at normal plasma concentrations. Cryoprecipitate consists of the cryoproteins recovered from FFP when it is rapidly frozen and then allowed to thaw at 2° to 6° C. These cryoproteins include fibrinogen, factor VIII, von Willebrand factor, factor XIII, and fibronectin. (Answer: B—Leukocyte reduction reduces febrile transfusion reactions)

37. A 58-year-old man with acute myelogenous leukemia received chemotherapy 10 days ago. He now presents to the emergency department with severe fatigue and shortness of breath. He has had no fever. Results of complete blood count are as follows: white cell count, 800/µl; hemoglobin level, 7.5 g/dl; platelet count, 43,000/µl.

Which of the following statements regarding indications for transfusion of blood products is true?
A. In patients with acute blood loss, the first treatment goal is transfusion of packed red blood cells
B. Platelet transfusions are contraindicated in autoimmune thrombocytopenia
C. The prevalence of bleeding increases significantly below a threshold of about 10,000 platelets/µl in otherwise asymptomatic patients
D. In chronically anemic patients, red cell 2,3-diphosphoglycerate production is decreased to maximize the red blood cells’ oxygen affinity

Key Concept/Objective: To know the indications for transfusion of blood products

The decision whether to use red cells depends on the etiology and duration of the anemia, the rate of change of the anemia, and assessment of the patient’s ability to compensate for the diminished capacity to carry oxygen that results from the decrease in red cell mass. Restoration of intravascular volume, usually with crystalloid, ensures adequate perfusion of peripheral tissue and is the first treatment goal for a patient with acute blood loss. In general, the decision to transfuse platelets rests on the answers to two questions: (1) Is the thrombocytopenia the result of underproduction or increased consumption of plateletsand (2) Do the existing platelets function normallyThrombocytopenia can result from decreased production caused by marrow hypoplasia or from increased consumption caused by conditions such as idiopathic thrombocytopenic purpura (ITP). Studies have shown that the prevalence of bleeding increases significantly below a threshold of about
10,000 platelets/µl in otherwise asymptomatic patients. Transfusion is appropriate in a bleeding patient whose platelet count is adequate but whose platelets are nonfunctional as a result of medications such as aspirin or nonsteroidal anti-inflammatory drugs or as a result of bypass surgery. Proper investigation of the causes of thrombocytopenia will identify clinical situations in which platelets should be withheld because they contribute to evolution of the illness. These disorders include thrombotic microangiopathies such as thrombotic thrombocytopenic purpura, hemolytic-uremic syndrome, and the HELLP syndrome (hemolysis, elevated liver enzymes, and a low platelet count). Platelet transfusions will not help patients with autoimmune thrombocytopenia (e.g., ITP), but they also will not harm them. (Answer: C—The prevalence of bleeding increases significantly below a threshold of about 10,000 platelets/µl in otherwise asymptomatic patients)

38. A 33-year-old white man presents with an exacerbation of Crohn disease, which is manifested by brightred blood from the rectum; abdominal pain; and anemia. You begin therapy for exacerbation of Crohn disease, and you also order the transfusion of 2 units of red blood cells. Approximately 30 minutes after the first unit of red cells is begun, the nurse calls and says the patient has a fever and “doesn’t feel well.”

Which of the following statements regarding transfusion complications is true?
A. Immediate hemolytic reactions are the result of an anamnestic response to an antigen to which the recipient is already sensitized
B. Delayed hemolytic reactions occur during primary sensitization and can be as severe as immediate hemolytic reactions
C. Until the cause of the hemolytic transfusion reaction is identified, the patient may only receive type O red cells or AB plasma
D. Fever without signs of hemolysis can be managed with acetaminophen; no further laboratory workup is necessary

Key Concept/Objective: To understand the potential complications of transfusions

Hemolytic transfusion reactions are classified as immediate or delayed, depending on their pathophysiology. Immediate hemolytic reactions are the result of a preexisting antibody in the recipient that was not detected during pretransfusion testing. Delayed hemolytic reactions are the result of an anamnestic response to an antigen to which the recipient is already sensitized. Clinical evidence of hemolysis is likely to be more severe in immediate hemolytic reactions and may include back pain, pain along the vein into which the blood is being transfused, changes in vital signs, evidence of acute renal failure, and signs of developing disseminated intravascular coagulation. Until the antibody causing the immune hemolysis is identified, only type O red cells and AB plasma should be used. Febrile reactions are characterized by the development of fever during transfusion or within 5 hours after transfusion. The differential diagnosis for a patient undergoing a nonhemolytic febrile transfusion reaction should always include unrecognized sepsis. When febrile reaction is suspected, immediate management consists of discontinuing the transfusion, obtaining appropriate cultures, and returning the product to the blood bank. (Answer: C—Until the cause of the hemolytic transfusion reaction is identified, the patient may only receive type O red cells or AB plasma)

39. A 65-year-old man presents to you for preoperative workup before undergoing aortic valve replacement for aortic regurgitation (indicated because of progressive left ventricular dysfunction, as revealed on echocardiogram) and coronary artery bypass surgery. He is interested in autologous blood donation. He has had chronic stable angina for the past 2 years, which is brought on by maximal exertion; his angina has remained unchanged for 1 year. For the past 2 days he has had increased urgency for urination and dysuria. On physical examination, he has a 2/4 diastolic murmur and suprapubic tenderness; otherwise, his examination is normal.

What absolute contraindication to autologous blood donation does this man have?
A. Angina
B. Aortic regurgitation
C. Active bacterial infection
D. Age older than 60 years

Key Concept/Objective: To know the absolute contraindications to autologous blood donation

This patient appears to have a UTI, so he cannot donate blood until that is resolved. Active bacterial infection is one of the three absolute contraindications to autologous blood donation; the other two are tight aortic stenosis and unstable angina. Although this man’s stable angina might temper one’s willingness to recommend autologous blood donation (especially because the risk of disease transmission in donated blood is low enough to make potential clerical error in the transfusion of autologous blood more of a concern), it is not an absolute contraindication, nor is aortic regurgitation or his age. (Answer: C—Active bacterial infection)

40. A 25-year-old woman who is 28 weeks pregnant is brought to the emergency department after an automobile collision. She complains of abdominal pain; her blood pressure is 85/60; and her pulse is 130. Normal fetal heart activity is found on fetal monitoring. Abdominal ultrasound reveals free fluid in the peritoneum.

Which of the following is the appropriate transfusion therapy for this patient

A. Whole blood
B. Packed red cells
C. Irradiated red cells
D. Leukocyte-reduced red cells

Key Concept/Objective: To know the appropriate choice for transfusion of red cells in a pregnant woman

This question highlights the concern about transmission of cytomegalovirus (CMV) during pregnancy. Whole blood, packed red cells, and irradiated red cells all carry the risk of CMV transmission. Blood from a CMV-negative donor is another choice, but because the prevalence of CMV infection varies widely from region to region in the United States, it is not always available. Use of cellular blood components that contain fewer than 5 × 106 leukocytes is effective in preventing the transmission of CMV. (Answer: D—Leukocyte-reduced red cells)

41. A 22-year-old man with hemophilia A is going to have impacted molars extracted. He has a history of prolonged bleeding after minor surgeries.

Which of the following is the most appropriate transfusion therapy for this patient during his dental procedure?
A. Cryoprecipitate
B. Factor VIII concentrate
C. Fresh frozen plasma
D. Platelets

Key Concept/Objective: To know the most appropriate transfusion support therapy for a patient with hemophilia A who is undergoing surgery in which there is a possibility of major bleeding

Factor VIII concentrate is the most appropriate of the choices because its means of preparation minimizes the risk of transmission of blood-borne infections to recipients. Cryoprecipitate is another option, but because it relies on the pooling of blood products from multiple donors, the risk of infection with blood-borne pathogens is much increased. Fresh frozen plasma does contain factor VIII, but only in low concentrations so that a much greater transfusion volume would be required. Platelet transfusion does not help to correct the underlying disorder. (Answer: B—Factor VIII concentrate)

42. Which of the following patients absolutely requires platelet transfusion?
A. A patient who has been taking aspirin for a headache and who is now scheduled for emergent evacuation of his subdural hemorrhage; platelet count, 100,000/µl
B. A patient with idiopathic thrombocytopenia; platelet count, 10,000/µl
C. A patient with thrombotic thrombocytopenic purpura; platelet count,
9,000/µl
D. A patient with end-stage liver disease who is complaining of easy bruising; platelet count, 50,000/µl

Key Concept/Objective: To know the indications for platelet transfusion

Although a platelet count of 100,000/µl is adequate for major surgery, because of this patient’s use of aspirin, those platelets are nonfunctional, and so platelet transfusion would be required. Although the platelet count of a patient with idiopathic thrombocytopenia is very close to the “trigger level” for platelet transfusion for an asymptomatic patient (< 10,000/µl), platelet transfusion is not absolutely required, especially owing to the fact that the platelets in these patients tend to function very well. Platelet transfusion is contraindicated in patients with thrombotic thrombocytopenic purpura; in such patients, plasmapheresis with fresh frozen plasma is indicated. In a patient with end-stage liver disease who has a platelet count of 50,000/µl, mild bleeding (easy bruising) is not an indication for platelet transfusion. (Answer: A—A patient who has been taking aspirin for a headache and who is now scheduled for emergent evacuation of his subdural hemorrhage; platelet count, 100,000/µl)

43. A 63-year-old multiparous woman is receiving packed red cells to treat symptomatic anemia after hip replacement surgery. Fifteen minutes into the transfusion, she has rigors. On physical examination, she appears anxious and diaphoretic; her temperature is 102.2° F (39° C); the rest of her examination is normal.

What is the first step in the diagnosis and management of this transfusion reaction?
A. Administer acetaminophen or meperidine for symptomatic relief
B. Draw blood for culturing
C. Stop the transfusion
D. Send the untransfused blood back to the blood bank for analysis

Key Concept/Objective: To understand the management of febrile transfusion reactions

The most important first step in managing febrile transfusion reactions is to stop the infusion immediately. Because bacterial infection can be a complication of transfusion or surgery, drawing blood for culturing is indicated but would not be the first step. Acetaminophen or, if the rigors are particularly severe, meperidine is helpful in the management of febrile transfusion reaction but should be preceded by discontinuance of the infusion. Sending the untransfused blood back to the blood bank is important so that the blood bank can obtain cultures from the product and verify that there have not been any errors in its production. (Answer: C—Stop the transfusion)

For more information, see Churchill WH: 5 Hematology: X Transfusion Therapy. ACP Medicine Online (www.acpmedicine.com). Dale DC, Federman DD, Eds. WebMD Inc., New York, October 2004

Hematopoietic Cell Transplantation

44. A 42-year-old white woman presented 2 months ago with menorrhagia. She was noted to be pancytopenic on initial laboratory evaluation. After an exhaustive workup, a diagnosis of aplastic anemia was made. The patient is being considered for hematopoietic cell transplantation.

Which of the following statements regarding hematopoietic cell transplantation is false?
A. Hematopoietic stem cells for transplantation may derive from bone marrow, peripheral blood, or umbilical cord blood
B. Syngeneic transplantations come from identical twins; identical twins are the best possible donors of stem cells
C. In allogeneic transplantations, if two persons do not share the same HLA antigens, B cells taken from one person will react vigorously to the mismatched HLA molecules on the surface of the cells from the other person
D. When compared with allogeneic transplantation, autologous transplantation has the advantage of avoiding graft versus host disease (GVHD) and associated complications

Key Concept/Objective: To understand the different types of hematopoietic stem cell transplantations

Hematopoietic stem cell transplantation can be categorized according to the relation between the donor and the recipient and according to the anatomic source of the stem cell. Hematopoietic stem cells for transplantation may derive from bone marrow, peripheral blood, or umbilical cord blood and may be harvested from a syngeneic, allogeneic, or autologous donor. Identical twins are the best possible donors of stem cells. When syngeneic donors are used, neither graft rejection nor GVHD will develop in the recipient. Only about one in 100 patients undergoing transplantation will have an identical twin. Allogeneic transplantation, which involves a related or unrelated donor, is more complicated than syngeneic or autologous transplantation because of immunologic differences between donor and host. With allogeneic hematopoietic cell transplantation, in which the immune system of the patient is provided by the graft, the clinical concerns are not only with the prevention of graft rejection by host cells surviving the pretransplant preparative regimen but also with the prevention of donor cells from causing immune-mediated injury to the patient (i.e., GVHD). Immunologic reactivity between donor and host is largely mediated by immunocompetent cells that react with HLAs, which are encoded by genes of the major histocompatibility complex. HLA molecules display both exogenous peptides (for example, from an infecting virus) and endogenous peptides, presenting them to T cells, an important step in the initiation of an immune response. If two persons do not share the same HLA antigens, T cells taken from one person will react vigorously to the mismatched HLA molecules on the surface of the cells from the other. These are reactions against so-called major HLA determinants. When compared with allogeneic transplantation, autologous transplantation has the advantage of avoiding GVHD and associated complications; disadvantages are that the autologous cells lack the antitumor effect of an infusion of allogeneic leukocytes (the so-called graft versus tumor effect) and may contain viable tumor cells. (Answer: C—In allogeneic transplantations, if two persons do not share the same HLA antigens, B cells taken from one person will react vigorously to the mismatched HLA molecules on the surface of the cells from the other person)

45. A 20-year-old African-American patient with sickle cell disease was recently evaluated for hematopoietic stem cell transplantation by his hematologist. He comes in to see you and is excited about the possibilities of cure but is concerned about possible complications of transplantations.

Which of the following statements regarding transplant complications is true?
A. Late toxicity (occurring weeks to months after transplantation) is usually the result of the preparative regimen and can include nausea, vomiting, skin rash, mucositis, and alopecia
B. Graft failure that occurs after autologous transplantation can result from marrow damage before harvesting, during ex vivo treatment, during storage, after exposure to myelotoxic agents, or as a result of infections with cytomegalovirus (CMV) or human herpesvirus type 6 (HHV-6)
C. Treatment of graft failure requires the use of higher doses of myelosuppressive agents
D. Veno-occlusive disease of the liver usually occurs after the first year and only rarely occurs in the subacute setting

Key Concept/Objective: To understand the complications of hematopoietic stem cell transplantation

Pretransplant preparative regimens are associated with a substantial array of toxicities, which vary considerably depending on the specific regimen used. For example, after the standard cyclophosphamide–total body irradiation regimen, nausea, vomiting, and mild skin erythema develop immediately in almost all patients. Oral mucositis inevitably develops about 5 to 7 days after transplantation and usually requires narcotic analgesia. By 10 days after transplantation, complete alopecia and profound granulocytopenia have developed in most patients. Veno-occlusive disease of the liver (also referred to as sinusoidal obstruction syndrome) is a serious complication of high-dose chemoradiotherapy; it develops in approximately 10% to 20% of patients. Veno-occlusive disease of the liver, characterized by the development of ascites, tender hepatomegaly, jaundice, and fluid retention, may occur at any time during the first month after transplantation; the peak incidence occurs at around day 16. Approximately 30% of patients who develop veno-occlusive disease of the liver die as a result of the disease, with progressive hepatic failure leading to a terminal hepatorenal syndrome. Although complete and sustained engraftment is the general rule after transplantation, in some cases marrow function does not return; in other cases, after temporary engraftment, marrow function is lost. Graft failure after autologous transplantation can result from marrow damage before harvesting, during ex vivo treatment, during storage, or after exposure to myelotoxic agents after transplantation. Infections with CMV or HHV-6 may also result in poor marrow function. Graft failure after allogeneic transplantation may be the result of immunologically mediated graft rejection and is more common after conditioning regimens that are less immunosuppressive, in recipients of T cell–depleted marrow, and in recipients of HLA-mismatched marrow. The treatment of graft failure begins with removal of all potentially myelosuppressive agents. (Answer: B—Graft failure that occurs after autologous transplantation can result from marrow damage before harvesting, during ex vivo treatment, during storage, after exposure to myelotoxic agents, or as a result of infections with cytomegalovirus [CMV] or human herpesvirus type 6 [HHV-6])

46. A 49-year-old woman is admitted to the hospital for weight loss, fatigue, and night sweats. A CBC ordered in the emergency department revealed anemia, thrombocytosis, and pronounced leukocytosis with a relatively normal differential consistent with a myeloproliferative disorder. You are concerned that her symptoms may be caused by a hematologic malignancy.

Which of the following statements regarding hematopoietic stem cell transplantation for malignant disease is false?
A. Hematopoietic stem cell transplantation is first-line therapy for chronic lymphocytic leukemia (CLL) and has a cure rate of 80%
B. The best results with allogeneic transplantation for acute myeloid leukemia (AML) are obtained in patients undergoing transplantation in first remission
C. Allogeneic transplantation can cure 15% to 20% of patients with acute lymphocytic leukemia (ALL) who fail induction therapy or in whom chemotherapy-resistant disease develops
D. In CML, the best results from allogeneic transplants are obtained in patients who receive transplants within 1 year of diagnosis

Key Concept/Objective: To know the indications for hematopoietic stem cell transplantation in malignant diseases

Allogeneic marrow transplantation cures 15% to 20% of patients with AML who fail induction therapy; indeed, it is the only form of therapy that can cure such patients. Thus, all patients 55 years of age or younger with newly diagnosed AML should have their HLA type determined, as should their families, soon after diagnosis to enable transplantation for those who fail induction therapy. The best results with allogeneic transplantation for AML are obtained in patients undergoing transplantation in first remission, in whom a cure rate of 40% to 70% is reported. As with AML, allogeneic transplantation can cure 15% to 20% of patients with ALL who fail induction therapy or in whom chemotherapy-resistant disease develops; thus, these patients are candidates for the procedure. Allogeneic and syngeneic marrow transplantations are the only forms of therapy known to cure CML. Time from diagnosis influences the outcome of transplantation during the chronic phase. The best results are obtained in patients who receive transplants within 1 year of diagnosis; progressively worse results are seen the longer the procedure is delayed. Use of marrow transplantation in CLL has received only limited attention, probably because of the indolent nature of the disease and its propensity to occur in older patients. (Answer: A—Hematopoietic stem cell transplantation is first-line therapy for chronic lymphoctic leukemia [CLL] and has a cure rate of 80%)

47. You are called to see a 26-year-old man in the hematology-oncology service because of fever and a low WBC count. He recently underwent induction chemotherapy for acute myelogenous leukemia. Yesterday, his absolute neutrophil count (ANC) was 500/mm3, and today it is 100/mm3. He has been anemic and thrombocytopenic but has not required transfusion. This morning, he developed a fever of 103.1° F (39.5° F). He has no focal central nervous sytem, respiratory, gastrointestinal, or urinary complaints other than severe stomatitis, caused by the chemotherapy. A careful physical examination fails to reveal any source of infection. Chest x-ray, blood and urine cultures, and a repeat complete blood count are ordered.

Which of the following would you recommend for this patient at this time?
A. No treatment until culture results are known
B. Lumbar puncture
C. Intravenous gentamicin and piperacillin
D. Oral ciprofloxacin
E. Administration of granulocyte colony-stimulating factor (G-CSF) and granulocyte-macrophage colony-stimulating factor (GM-CSF)

Key Concept/Objective: To know the indications for empirical treatment of febrile neutropenia and the best antibiotic combination

Febrile neutropenia (ANC < 500/mm3) is an urgent indication for careful history and physical examination, expedient collection of cultures, expedient use of radiography (e.g., chest x-ray), and initiation of empirical antibiotics. In febrile neutropenic patients, the most common sources of infection are the lungs, the genitourinary system, the GI tract, the oropharynx, and the skin. Initially, the infecting organisms are the usual flora or are infecting agents commonly found at the anatomic site of infection. However, in patients with recurrent infections or those who require prolonged courses of antibiotics, unusual organisms can be responsible for the infection. Frequently, the usual signs and symptoms of infection are attenuated or absent in these patients because of the absence of the inflammatory responses to infection. In the neutropenic patient, minor infections that might otherwise have been well localized can become serious disseminated infections very quickly. Management includes careful evaluation of the oropharynx, skin, lungs, GI tract, and genitourinary tract for subtle signs of infection. Cultures and a chest x-ray are obtained, and empirical antibiotics are started. Clinicians can select traditional combinations of a ß-lactam antibiotic active against Pseudomonas (e.g., piperacillin) and an aminoglycoside (e.g., gentamicin or tobramycin). Although colony-stimulating factors may be considered for adjunctive use in selected high-risk, severely ill neutropenic patients, they are not indicated in most febrile neutropenic patients. (Answer: C—Intravenous gentamicin and piperacillin)

48. A 23-year-old woman underwent allogeneic bone marrow transplantation for acute myelogenous leukemia. On day 11, she began to complain of right upper quadrant pain, and her weight began to climb. On examination, peripheral edema and tender hepatomegaly were appreciated. Skin examination was unremarkable. Laboratory testing revealed a bilirubin level of 2.4 mg/dl and an alanine aminotransferase level of 146 U/L. Over the next several days, she developed increasing abdominal girth, and her bilirubin level increased to 12 mg/dl.

What is the most likely diagnosis for this patient?
A. Graft versus host disease (GVHD)
B. Acute hepatitis A infection
C. Acute cytomegalovirus infection
D. Graft rejection
E. Veno-occlusive disease of the liver

Key Concept/Objective: To recognize veno-occlusive disease as a potential complication of hematopoietic stem cell transplantation

This patient presents with typical findings of veno-occlusive disease, including ascites, hepatomegaly, jaundice, and fluid retention. Veno-occlusive disease typically occurs in the first few weeks after transplantation. Pathologically, there is cytotoxic injury to the hepatic venulae and sinusoidal endothelium, resulting in vascular blockage (the clinical picture is similar to that of Budd-Chiari syndrome). There is a high mortality, and research continues in the fields of treatment and prevention. Other possible causes include GVHD, viral hepatitis, drug reaction, sepsis, heart failure, and tumor invasion. However, acute ascites and fluid retention are more typical of veno-occlusive disease. (Answer: E—Veno-occlusive disease of the liver)

49. A 42-year-old man presents with fatigue and progressive left upper quadrant pain. On examination, the spleen is palpable. CBC reveals a hematocrit of 32% and a WBC count of 97,000/mm3. Bone marrow biopsy reveals hyperplasia of the granulocytic series, and the cytogenetic analysis confirms the presence of the Philadelphia chromosome. A diagnosis of chronic myeloid leukemia (CML) is made.

What is the most appropriate step to take next for this patient?
A. Search for HLA-matched sibling
B. Initiate busulfan therapy
C. Initiate interferon-alfa therapy
D. Initiate hydroxyurea therapy
E. Leukapheresis

Key Concept/Objective: To understand the role of allogeneic transplantation in chronic myelogenous leukemia

If an appropriate HLA-matched sibling can be found, most experts would recommend allogeneic transplantation as initial therapy for a person younger than 50 years who is diagnosed with CML. In older patients, therapy with interferon alfa or hydroxyurea may be more appropriate. Therefore, the search for potential donors is the critical first step. Leukapheresis is only infrequently used (primarily in chronic lymphocyte leukemia) when white cell counts are extremely elevated and there are acute CNS symptoms. (Answer: A— Search for HLA-matched sibling)

50. What is the chance that the sister of the patient in Question 48 will be an HLA match?
A. 0%
B. 10%
C. 25%
D. 50%
E. 75%

Key Concept/Objective: To know the odds of finding an appropriate donor in a transplant candidate

The genes that encode HLA class I and II antigens are tightly linked and are typically inherited together. Therefore, there is a one-in-four chance of inheriting the identical haplotype of a sibling. (Answer: C—25%)

51. An 18-year-old woman underwent allogeneic bone marrow transplantation for Hodgkin disease that had failed to respond to first-line chemotherapy. Roughly 6 weeks after transplantation, she is diagnosed with acute GVHD.

Which of the following is not typically associated with GVHD?
A. Abdominal pain
B. Acute renal failure
C. Maculopapular rash
D. Diarrhea
E. Elevation of hepatic transaminase levels

Key Concept/Objective: To understand the clinical presentation of acute GVHD

GVHD is quite common in patients receiving allogeneic transplants. The most common manifestation is that of a maculopapular rash, frequently involving the palms and soles. The second most common organ affected is the liver. Symptoms of liver involvement typically include an elevation in levels of transaminase, alkaline phosphatase, and conjugated bilirubin; these changes are the result of damage to the small bile ducts. GI symptoms include diarrhea, anorexia, and crampy abdominal pain. Efforts to prevent GVHD include the use of immunosuppressive agents in the early posttransplantation period. GVHD is most frequently treated with glucocorticoids. (Answer: B—Acute renal failure)

52. A 43-year-old man with CML is being evaluated for allogeneic bone marrow transplantation. On questioning, he states that in the past, he had an allergic rash to trimethoprim-sulfamethoxazole.

What would be the most appropriate regimen for Pneumocystis carinii prophylaxis for this patient?
A. Dapsone
B. Atovaquone
C. Attempted desensitization to sulfa before transplantation
D. Intravenous pentamidine
E. Aerosolized pentamidine

Key Concept/Objective: To appreciate the superiority of sulfa in preventing P. carinii infection in the transplant patient

P. carinii once caused pneumonia in up to 10% of transplant patients. The risk of this complication is nearly eliminated through the use of appropriate prophylaxis. Trimethoprimsulfamethoxazole given 1 week before transplantation and then twice weekly after engraftment is very effective. The other agents listed as choices are all active against Pneumocystis, but trimethoprim-sulfamethoxazole is the most effective. Efforts should be made to desensitize the patient to sulfa. If these efforts are unsuccessful, dapsone is typically used. (Answer: C—Attempted desensitization to sulfa before transplantation)

For more information, see Appelbaum FR: 5 Hematology: XI Hematopoietic Cell Transplantation. ACP Medicine Online (www.acpmedicine.com). Dale DC, Federman DD, Eds. WebMD Inc., New York, April 2004

Hemostasis and Its Regulation

53. A newborn develops significant bleeding from the circumcision site. The family history indicates that one cousin has a bleeding disorder. The patient’s platelet count and morphology are normal; however, the bleeding time is very prolonged. A platelet function assay-100 (PFA-100) is abnormal; prothrombin time (PT) and partial thromboplastin time (PTT) are normal. Platelet aggregometry shows poor aggregation.

Which of the following molecules is most likely to be deficient in this newborn?
A. von Willebrand factor
B. Glycoprotein (GP) Ib-IX-V
C. GPIIb-IIIa
D. Tissue plasminogen activator

Key Concept/Objective: To understand the mechanisms of platelet activation

Platelets are activated at the site of vascular injury to form a plug to stop bleeding. Platelet activation involves four distinct processes: adhesion, aggregation, secretion, and procoagulant activity. Platelet adhesion is primarily mediated by the binding of GPIb-IX-V complex to von Willebrand protein. Aggregation involves binding of fibrinogen to the platelet fibrinogen receptor GPIIb-IIIa. Congenital deficiency of GPIIb-IIIa or fibrinogen leads to Glanzmann thrombasthenia and afibrinogenemia. The GPIIb-IIIa fibrinogen pathway is the final common course for platelet aggregation. Platelet protein secretion occurs after platelet stimulation, with the release of granules containing serotonin and adenosine diphosphate (ADP), which stimulate and recruit more platelets. Platelet procoagulation involves the assembly of the enzyme complexes in the clotting cascade on the platelet surface. Tissue plasminogen activator is a fibrinolytic factor; its deficiency causes a hypercoagulable state. (Answer: C—GPIIb-IIIa)

54. A 42-year-old man with advanced AIDS presents to a walk-in clinic complaining of leg pain that started
4 days ago. The patient has no history of deep vein thrombosis (DVT) and no family history of DVT. He denies experiencing any recent trauma, fractures, or surgeries. On physical examination, the patient looks chronically ill. His left leg has moderate edema and tenderness to palpation. Compression ultrasonography shows a proximal DVT. His platelet count, PT, and PTT are within normal limits.

Which of the following is the most likely cause of this patient’s hypercoagulable state?
A. Antiphospholipid syndrome
B. Decreased levels of free protein S
C. Decreased levels of factor XIII
D. Congenital protein S deficiency

Key Concept/Objectives: To understand the mechanisms for acquired protein S deficiency

Antithrombin III, protein C, and protein S are important components of the control mechanisms that modulate coagulation. Protein S circulates in two forms: a free form, in which it is active as an anticoagulant; and a bound, inactive form, in which it is complexed to C4b-binding protein of the complement system. C4b-binding protein acts as an acute phase reactant. The resultant increase in inflammatory state reduces the activity of free protein S, enhancing the likelihood of thrombosis. In this patient, advanced HIV disease is causing an inflammatory state in which the levels of free functional protein S are decreased. Antiphospholipid syndrome can be a cause of a hypercoagulable state; commonly seen laboratory abnormalities are thrombocytopenia and a prolonged PTT secondary to the presence of an inhibitor. Factor XIII deficiency causes a bleeding disorder. (Answer: B—Decreased levels of free protein S)

55. A 37-year-old woman was scheduled to undergo elective cholecystectomy. As part of her preoperative evaluation, her surgeon ordered an assessment of bleeding time, which showed that bleeding time was prolonged. The patient has now been referred to you for evaluation. She remembers having one episode of moderate bleeding after a tooth extraction a few years ago. Her father had a history of mild to moderate bleeding after surgical procedures. Her physical examination is unremarkable, and her platelet count is normal.

Which of the following tests would be appropriate for the initial evaluation of this patient?
A. Thrombin time
B. Assessment of factor VII levels
C. Assessment of factor XIII levels
D. PFA-100

Key Concept/Objective: To understand the uses of different coagulation tests

This patient has a prolonged bleeding time and a history of a previous bleeding episode. She also has a family history of a mild bleeding disorder. Von Willebrand disease is the most likely etiology. The testing of bleeding time primarily measures platelet function. A prolonged bleeding time with a platelet count over 100,000/µl suggests impaired platelet function. The bleeding time is difficult to standardize, and a normal bleeding time does not predict the safety of a surgical procedure. Bleeding time should not be used as a general screening test in a preoperative setting. Although once used commonly for screening of platelet disorders, bleeding time has been replaced by the PFA-100. PFA-100 is a newly developed automated test of platelet function. Citrated blood from the patient is aspirated onto a membrane coated with collagen and epinephrine or collagen and ADP in which a central aperture is made. The time it takes for blood flow through the membrane to stop is denoted as the closure time and is a measure of platelet function. The closure time is prolonged in patients with von Willebrand disease or other platelet functional defects. PFA-
100 should be considered the first-line test for platelet function disorders. Thrombin time is used to test for abnormalities of the conversion of fibrinogen to fibrin. Thrombin time is prolonged in patients with severe liver disease and DIC and those undergoing heparin therapy. Factor VII levels are measured in patients who have a prolonged PT; it is a test of the extrinsic system. Factor XIII is the only clotting factor whose activity is not assessed in PT or PTT; a deficiency should be suspected in an infant who experiences bleeding after circumcision or in an adult with unexplained bleeding. (Answer: D—PFA-100)

56. A 25-year-old man comes to your clinic for follow-up after being discharged from a local hospital, where he presented with a DVT. He did not have any previous episodes of DVT, and he denied having any obvious precipitating factor. He says his older brother was diagnosed with a DVT 1 year ago. At the hospital, the patient was started on heparin and warfarin as an inpatient; he was discharged on warfarin, with instructions to take it for 6 months. The discharge summary from the hospital contains some laboratory information from blood obtained at the moment of discharge. His PTT was slightly prolonged; his INR was 2.2; his protein C levels were decreased at 70% of normal; and his protein S levels were decreased at
40% of normal.

On the basis of this information, which of the following is the most likely diagnosis for this patient?
A. No diagnosis can be made on the basis of this information
B. Antiphospholipid syndrome
C. Protein S deficiency
D. Combined protein S and protein C deficiencies

Key Concept/Objective: To understand the appropriate timing of tests for inhibitors of hemostasis

This patient is a young man with a DVT and a family history of DVT. He likely has a hereditary condition causing a hypercoagulable state. Levels of protein C and protein S were obtained to assess for the possibility of an occult hypercoagulable state. Levels of protein C and protein S can be obtained by functional and immunologic methods. Because protein C and protein S are vitamin K dependent, their measurement can be problematic in patients taking warfarin. It is best to measure protein C or protein S when the patient has been off warfarin for 3 to 4 weeks. The low values seen in this patient could be explained by the use of warfarin; in addition, the formation of a clot will per se cause a decrease in both protein C and protein S levels. A prolonged PTT can be caused by a clotting factor deficiency or an inhibitor. Antiphospholipid syndrome can cause an inhibitor that can be associated with a hypercoagulable state. A mixing study will still show the PTT to be prolonged if an inhibitor is present. The most likely explanation of the slightly prolonged PTT in this patient is the patient’s receiving heparin as an inpatient, just before the blood sample was obtained. (Answer: A—No diagnosis can be made on the basis of this information)

For more information, see Leung LLK: 5 Hematology: XII Hemostasis and its Regulation. ACP Medicine Online (www.acpmedicine.com). Dale DC, Federman DD, Eds. WebMD Inc., New York, June 2003

Hemorrhagic Disorders

57. A 25-year-old woman presents for routine examination. She has been well, but she has a long history of having heavier menstrual periods than other women she knows. This has not caused her any major problems. On review of systems, she notes a history of easy bleeding after dental work. Physical examination is normal except for petechiae on her shins. Results of coagulation studies are normal; her hematocrit is 32, with a platelet count of 56,000/µl. Your differential diagnosis includes idiopathic thrombocytopenic purpura (ITP).

Which of the following statements about ITP is false?
A. ITP is associated with splenomegaly
B. ITP is associated with HIV infection, acute viral illnesses, and some autoimmune diseases
C. Patients with platelet counts over 50,000/µl do not routinely require treatment
D. Treatment is indicated for patients with platelet counts below 20,000 to 30,000/µl or for those whose platelet count is less than 50,000/µl and who have risk factors for bleeding (including hypertension, peptic ulcer disease, or a vigorous lifestyle)
E. The disorder is relatively benign; the most common cause of death in adult patients with this disorder is intracranial bleeding

Key Concept/Objective: To be familiar with the presentation, course, associations, and platelet levels requiring treatment in cases of ITP

The presentation of ITP as a chronic illness with a mild bleeding diathesis is not uncommon in adults. Women are affected more frequently than men. In most adult patients (90%), ITP will follow a course characterized by chronic thrombocytopenia; in most children, platelet counts will return to normal within 3 months. ITP is infrequently associated with splenomegaly. In patients with this finding, other causes of thrombocytopenia, such as hypersplenism associated with liver disease, should be considered. Other diagnoses to be considered in patients with ITP and splenomegaly are lymphoma and systemic lupus erythematosus (SLE). Although most cases of ITP are idiopathic, it is well known that ITP can be associated with a number of underlying conditions, including HIV, viral illnesses (e.g., Epstein-Barr virus [EBV] infection), and autoimmune disease, most notably SLE. In adults, in contrast to children, ITP can present as a smoldering disease. Therapy is recommended for asymptomatic patients whose platelet counts are persistently lower than 20,000 to 30,000/µl and for patients with significant bleeding complications. Although in most patients ITP follows a relatively benign course, serious bleeding complications are occasionally seen. Of these bleeding complications, intracranial bleeding is the most dreaded because it is commonly fatal. Intracranial bleeding is more commonly seen when the platelet count drops below 10,000/µl; other risk factors include advanced age and concomitant medical illness. (Answer: A—ITP is associated with splenomegaly)

58. A 33-year-old woman is admitted to the hospital with altered mental status. Her physician calls you for consultation; he is concerned that the patient may have thrombotic thrombocytopenic purpura (TTP). He notes that she has anemia, thrombocytopenia, and a high fever and that she is disoriented. He asks your opinion regarding certain laboratory tests he has sent.

Which of the following findings is NOT consistent with a diagnosis of TTP?
A. Schistocytes and helmet cells seen on blood smear
B. A positive direct Coombs test
C. Normal results on coagulation testing
D. An elevated serum LDH level
E. Neutrophilic leukocytosis

Key Concept/Objective: To be able to recognize the laboratory abnormalities associated with TTP

Patients with TTP typically present with the following pentad of signs and symptoms: (1) thrombocytopenia; (2) microangiopathic hemolytic anemia with schistocytes and helmet cells; (3) renal dysfunction, which is usually mild; (4) fever, which can be very high; and (5) neurologic symptoms, including seizures and a clouded sensorium. TTP is not associated with a positive direct Coombs test; hemolysis is not immune mediated. This finding should make one consider autoimmune hemolytic anemias (such as those associated with Evan syndrome and SLE, both of which should be considered in the differential diagnosis). The other findings are consistent with TTP and are helpful in establishing the diagnosis and differentiating it from other causes of anemia and thrombocytopenia (such as disseminated intravascular coagulation). Prompt plasmapheresis should be instituted in patients with TTP. (Answer: B—A positive direct Coombs test)

59. A dentist wishes to learn more about von Willebrand disease (vWD); he recently had a patient who has the disorder suffer excessive bleeding after dental work. In your discussion, you outline several key facts about vWD, including how these patients may require treatment before dental procedures and other surgeries.

Which of the following statements is true about vWD?
A. Results of coagulation studies are normal in patients with vWD
B. Patients with vWD have thrombocytopenia
C. Most cases are the result of impaired function of von Willebrand factor (vWF)
D. An abnormal result on the ristocetin-induced platelet aggregation test is consistent with abnormal function of the patient’s vWF (type 2 vWD)
E. It affects very few people

Key Concept/Objective: To know some common facts about vWD

vWD is the most common hereditary bleeding disorder. It may be the result of low levels of vWF (type I vWD, which accounts for 75% of cases); qualitative defects in vWF function (type II vWD), which can be determined by use of the ristocetin-induced platelet aggregation test; or a severe or total deficiency of vWF (type 3 vWD, the least common form). vWF acts as a carrier molecule for factor VIII and is important in the stability and half-life of factor VIII; therefore, vWD is associated with a prolonged activated partial thromboplastin time. The disorders of platelet function associated with vWD are not associated with decreases in platelet count. (Answer: D—An abnormal result on the ristocetin-induced platelet aggregation test is consistent with abnormal function of the patient’s vWF [type 2 vWD])

60. A 7-year-old boy with advanced pancreatic adenocarcinoma is diagnosed with Trousseau syndrome (chronic disseminated intravascular coagulation [DIC] secondary to malignancy). Approximately 2 months ago, he was found to have a deep vein thrombosis (DVT).

Which of the following is seen in cases of chronic (compensated) DIC?
A. Thrombocytopenia
B. Elevated coagulation studies
C. Elevated D-dimer level
D. Predisposition toward bleeding rather than thrombosis

Key Concept/Objective: To be familiar with the common laboratory and clinical findings of chronic (compensated) DIC

Trousseau syndrome is a chronic form of DIC. When onset of DIC is slow, as is seen in some patients with cancer, compensation can occur. In these patients, a procoagulant state exists, and the patient has a predisposition for arterial and venous thrombosis (unlike in the acute forms of DIC, in which bleeding predominates). The typical laboratory abnormalities associated with acute DIC (thrombocytopenia, elevations in levels of coagulation factors, microangiopathic hemolytic anemia, decreased fibrinogen levels, and elevated Ddimer levels) are usually not present in patients with the chronic form; the exception to this is an elevated D-dimer level. Therapy is directed at the underlying disease. This patient with Trousseau syndrome and DVT should receive subcutaneous heparin if its use is not contraindicated. (Answer: C—Elevated D-dimer level)

For more information, see Leung LLK: 5 Hematology: XIII Hemorrhagic Disorders. ACP Medicine Online (www.acpmedicine.com). Dale DC, Federman DD, Eds. WebMD Inc., New York, January 2002

Thrombotic Disorders

61. A 38-year-old woman develops a deep vein thrombosis (DVT) in her left leg during a cross-country car trip. She is treated with heparin for 3 days, and she is started on warfarin. She is discharged from the hospital with an international normalized ratio (INR) of 2.5. Three days later, she presents with severe pain in her left hand. Examination reveals an infarction of the third digit, with severe pain, purpura, and erythema. Her INR is 2.4. She has been compliant with regard to her medication regimen.

Which of the following is most consistent with this patient’s disease process?
A. Protein S deficiency
B. Protein C deficiency
C. Hyperhomocysteinemia
D. Antithrombin III (AT-III) deficiency

Key Concept/Objective: To understand the diagnosis of protein C deficiency

Protein C deficiency results in a loss of ability to inactivate factor VIIa and factor Va, two major cofactors that regulate the clotting cascade. Warfarin lowers protein C levels as well as the levels of all the vitamin K-dependent clotting factors. Because the half-lives of factor Xa and prothrombin are longer than that of protein C, initiation of warfarin therapy can induce a paradoxical state of hypercoagulability. This patient likely has a heterozygous protein C deficiency that was uncovered when she was treated with warfarin. In patients in whom protein C deficiency is suspected, heparin and warfarin should be initiated concomitantly. Heparin therapy is initially started on an inpatient basis; warfarin is indicated for outpatient treatment. Another rare complication of warfarin therapy is skin necrosis, which can be severe. (Answer: B—Protein C deficiency)

62. A 58-year-old man with type 2 diabetes mellitus, hypertension, and coronary artery disease presents to your clinic for pain and swelling of his left leg. He states that his symptoms have been progressively worsening for 3 days and that he has now developed pain with ambulation. He denies having fever, shortness of breath, palpitations, or rash, but he adds that he was admitted to the hospital 10 days ago for chest pain. He states that he was treated with “medicine” and was told that he had had a small heart attack. On physical examination, the circumference of his left calf is 49 cm; the circumference of his right calf is 45 cm. The Homan sign is negative. On review of laboratory data, the complete blood count (CBC) is normal except for the platelet count, which is 60,000/µl. In the past, his CBC has been normal. Doppler ultrasound reveals a left superficial femoral DVT.

Which of the following statements regarding this patient is true?
A. Heparin-induced thrombocytopenia (HIT) is not likely in this patient because he developed symptoms more than 5 days after treatment was initiated
B. DVT is the most common event leading to the diagnosis of HIT
C. Diagnosis of HIT should be made by heparin-induced platelet aggregation assay because of the high sensitivity of this test
D. The patient can be safely treated with warfarin and low-molecularweight heparin because of the low immunogenicity of this type of heparin

Key Concept/Objective: To understand heparin-induced thrombocytopenia

HIT is a relatively frequent drug reaction that can potentially cause life-threatening arterial and venous thrombosis. HIT typically develops 5 to 10 days after initiation of therapy; it can, however, develop up to 2 1 weeks afterward. In patients who received heparin within the previous 100 days and are being retreated, the onset can be rapid—within hours after starting heparin. HIT is generally defined as a platelet count below 150 × 109 or a drop in the platelet count by more than 50% of the pretreatment peak. Venous thrombosis is more common than arterial thrombosis; DVT is the most common event leading to diagnosis of HIT. Diagnosis of HIT should be based on history and clinical findings; treatment should be initiated before laboratory confirmation. The most widely used assay in the diagnosis of HIT is heparin-induced platelet aggregation; however, this test has a low sensitivity. Treatment of HIT involves discontinuing all forms of heparin, including the use of heparin in flushes of subcutaneous lines. Various anticoagulants have been approved, including danaparoid, hirudin, lepirudin, and argatroban. Low-molecular-weight heparin is not safe to use in patients with HIT because of its high cross-reactivity with standard heparin. (Answer: B—DVT is the most common event leading to the diagnosis of HIT)

63. A 24-year-old woman presents with a 1-day history of pain and swelling in her right leg. She had a DVT once before, when she was receiving oral contraceptives; she now takes no medications. On physical examination, the circumference of her right leg is increased. Ultrasound reveals a DVT in her thigh.

Which of the following tests would be helpful in the acute setting to determine the cause of her suspected hypercoagulable state?
A. Protein S level
B. Protein C level
C. Factor V Leiden
D. AT-III level

Key Concept/Objective: To understand the implications of timing on the workup of a hypercoagulable state

In this young woman with a history of DVTs, a hypercoagulable state should be suspected. In acute thrombosis, many clotting factor inhibitors are consumed, and therefore, an assessment of the levels of these inhibitors would not be useful. If plasma levels are high, it would be possible to argue that the patient does not have a hereditary deficiency; levels could be low secondary to the acute event or to an inherited cause. Some of the coagulation cascade inhibitors that are consumed immediately after a clotting event are protein C, protein S, and AT-III. Factor V Leiden mutations can be tested at any time. (Answer: C— Factor V Leiden)

64. A 44-year-old white woman presents with pain and swelling in her left lower extremity. Ultrasound reveals a left superficial femoral DVT. In the past, she experienced one other episode of DVT, for which she underwent treatment with warfarin for 6 months. She takes no medications except oral contraceptive pills. Her family history is significant in that, last year, a younger sister was diagnosed as having DVT.

Which of the following statements is true regarding the treatment of this patient?
A. She should again be treated with warfarin for 6 months
B. She should avoid the use of oral contraceptives
C. She should be tested for factor V Leiden after her anticoagulation therapy is terminated
D. All family members need to be screened for factor V Leiden

Key Concept/Objective: To understand the management of factor V Leiden mutations

Factor V Leiden, now considered the most common hereditary hypercoagulable state, is a defect caused by a mutated form of factor V that is resistant to the anticoagulation effects of activated protein C. In patients with thrombophilia, its prevalence is as high as 20% to 50%. The clinical manifestations are similar to other anticoagulant deficiencies and consist mainly of venous thrombosis. Management of factor V Leiden is also similar to that of other hypercoagulable states; warfarin is the mainstay of therapy. Patients with a first episode of thrombosis should undergo anticoagulation therapy for a period of 6 months. Patients with recurrent thromboses require lifelong therapy. Although the lifetime relative risk of thrombosis for patients with this deficiency is comparatively lower than that associated with protein C, protein S, and AT-III deficiencies, there is a synergistic relationship with the use of oral contraceptives. As such, women with factor V Leiden should avoid the use of oral contraceptives. Routine screening of family members is not warranted and is not cost-efficient. (Answer: B—She should avoid the use of oral contraceptives)

65. An 18-year-old man presented to the emergency department with complaints of shortness of breath. He reported that the symptoms came on suddenly 2 days ago and that they had been progressively worsening. He complained of left-sided chest pain that occurred when he took a deep breath; the pain did not radiate, nor was it associated with nausea, vomiting, or diaphoresis. He stated that he had had a bicycle accident 1 week earlier but that otherwise he had been doing well. On examination, the patient was tachypneic and tachycardic; his heart rate was 110 beats/min. With the patient receiving 2 L of oxygen by nasal cannula, the oxygen saturation was 94%. Physical examination revealed some crackles in the left base but was otherwise unremarkable. CBC and blood chemistries were within normal limits, and a chest x-ray was normal. Arterial blood gas measurements were made; the arterial oxygen tension (PaO2) was 60 mm Hg on room air. Ventilation-perfusion scanning showed a large right lower lobe perfusion defect, which was interpreted as indicating a high probability of pulmonary embolism. The patient underwent anticoagulation therapy for 6 months. At follow-up, with the patient off medication, a hypercoagulable workup is performed. Laboratory results are all within normal limits except for the AT-III level, which is low.

Which of the following statements regarding AT-III deficiency is false?
A. AT-III deficiency can be inherited or acquired
B. AT-III inactivates factor Xa and thrombin
C. AT-III deficiency causes severe arterial thrombosis
D. For patients with AT-III deficiency, the risk of thrombosis increases with age

Key Concept/Objective: To understand AT-III deficiency

AT-III deficiency is an autosomal dominant trait that affects nearly 1 in 2,000 people. There are two types of AT-III deficiency: inherited and acquired. The inherited form has two subsets: quantitative deficiency and qualitative deficiency. In some cases, AT-III deficiency may be acquired, as with disseminated intravascular coagulation or severe liver disease or through the administration of I.V. heparin. AT-III normally inactivates factor Xa and thrombin; patients with AT-III deficiency show evidence of continuous factor X activation and thrombin generation. The typical presentation of AT-III deficiency is similar to that of other hypercoagulable states. There is no evidence that AT-III deficiency increases the risk of arterial thrombosis. The two hypercoagulable states more closely related to arterial thrombosis are the antiphospholipid syndrome and hyperhomocystinemia. (Answer: C—AT-III deficiency causes severe arterial thrombosis)

66. A 26-year-old man presents with new-onset left lower extremity swelling and pain of 6 hours’ duration.
He takes no medications and has no history of trauma, immobilization, or prior thrombosis. His family history is remarkable for two “blood clots” in his mother. Compression ultrasonography confirms occlusive thrombus in the left superficial femoral vein.

Which of the following is the most appropriate sequence of interventions for this patient?
A. Start heparin and warfarin immediately, send tests for the hypercoagulable state before warfarin reaches therapeutic levels, and discontinue heparin after the international normalized ratio (INR) reaches therapeutic levels
B. Send tests for the hypercoagulable state, then start heparin and warfarin concurrently, and discontinue heparin after 5 days’ overlap
C. Send tests for the hypercoagulable state, then start heparin and warfarin concurrently, and discontinue heparin when the INR reaches therapeutic levels
D. Start heparin and warfarin immediately, discontinue heparin after 5 days’ overlap, and evaluate for the hypercoagulable state after warfarin therapy is completed
E. Evaluate for the hypercoagulable state, but no anticoagulation is indicated for superficial thrombophlebitis

Key Concept/Objective: To understand the timing of workup and duration of therapy for patients presenting with a new DVT

A 26-year-old man presenting with new-onset thrombosis and a positive family history is highly suspicious for a hereditary hypercoagulable state and should be worked up for this. Because the levels of protein C and antithrombin III can be diminished in the setting of acute thrombosis and because heparin and warfarin also alter these levels, the optimal time for the workup is after the patient has completed therapy. Exceptions to this rule include the antiphospholipid antibody syndrome, in which early diagnosis can affect therapy and disorders for which specific genotypic tests are available (e.g., factor V Leiden), which will be accurate at any time. Because the INR (prothrombin time) is heavily dependent on factor VII, which has a short half-life, it rises fairly quickly after warfarin is begun. However, therapeutic anticoagulation may take several days longer because of the persistence of factor X and prothrombin. Overlapping heparin and warfarin by 5 days is thought to limit the risk of propagation of thrombus caused by delayed therapeutic anticoagulation. The confusingly named superficial femoral vein is in fact in the deep system and warrants therapy. (Answer: D—Start heparin and warfarin immediately, discontinue heparin after 5 days’ overlap, and evaluate for the hypercoagulable state after warfarin therapy is completed)

67. A 58-year-old woman is 2 days’ status post–total hip replacement. She has been receiving subcutaneous heparin as prophylaxis for DVT. You are asked to see the patient to evaluate new-onset dyspnea. On examination, the patient is tachypneic, tachycardic, and diaphoretic. She is agitated and complains of substernal chest pain. Chest examination reveals few bibasilar crackles. Cardiac, abdominal, and extremity examinations are normal except for her surgical wounds, which seem to be healing well. Laboratory results are as follows: pH, 7.47; PaCO2, 31; PO2, 65; WBC, 11,500; Hb, 11.2; HCT, 33; platelets, 54,000/µl; prothrombin time, 14 sec; INR, 1.1; PTT, 44 sec (normal, 24–32; therapeutic range for heparin, 60–80); and chemistry panel, within normal limits. ECG reveals a 2 mm ST depression in leads II, III, and aVf; and chest x-ray is normal.

Which of the following is the best course of action for this patient at this point?

A. Bolus with I.V. heparin; begin heparin infusion; and arrange for urgent coronary angiography
B. Bolus with I.V. heparin; begin heparin infusion; and arrange for urgent ventilation-perfusion scan
C. Switch to low-molecular-weight heparin (LMWH) subcutaneously while arranging further diagnostic testing
D. Stop all heparin, and bolus with I.V. methylprednisolone and diphenhydramine for allergic reaction to heparin
E. Stop all heparin, and anticoagulate with hirudin while arranging further diagnostic testing

Key Concept/Objective: To understand heparin-induced thrombocytopenia and its therapy

This patient is status post-hip surgery and has been receiving heparin for 2 days. She now presents with a clinical scenario that suggests pulmonary embolism or myocardial ischemia, or both, as well as a platelet count of 54,000/µl. In this setting, one should be highly suspicious of heparin-induced thrombocytopenia, which can present with venous or arterial thrombosis. While the next diagnostic test is a matter of clinical judgment, the crucial first step is discontinuing all heparin, including I.V. flushes, and initiating anticoagulation with hirudin (a potent thrombin inhibitor derived from medicinal leeches). Other alternatives would be lepirudin or danaparoid. LMWH, unfortunately, is not considered safe in the setting of heparin-induced thrombocytopenia and would not be a good alternative in this setting. (Answer: E—Stop all heparin, and anticoagulate with hirudin while arranging further diagnostic testing)

68. A 68-year-old man presents with new onset of right-sided DVT without apparent risk factors. Therapy is initiated, and the possibility of underlying cancer is raised. You are consulted regarding appropriate evaluation for occult malignancy.

What would you recommend for this patient

A. Careful history, physical examination, routine blood counts and chemistries, chest x-ray (CXR), fecal occult blood testing (FOBT), and prostate-specific antigen (PSA); if these are not revealing, no further evaluation is necessary
B. Careful history, physical examination, routine blood counts and chemistries, CXR, FOBT, and PSA; if these are not revealing, proceed with colonoscopy
C. Careful history, physical examination, routine blood counts and chemistries, CXR, FOBT, and PSA; if these are not revealing, proceed with CT scan of the chest, abdomen, and pelvis
D. Careful history, physical examination, routine blood counts and chemistries, CXR, FOBT, and PSA; if these are not revealing, proceed with bone scan

Key Concept/Objective: To understand the malignancy workup in a patient presenting with newonset DVT

There is a documented association between malignancy and thrombosis; in a recent prospective trial, patients with idiopathic DVT had an 8% incidence of diagnosis of cancer in the following 2 years, with an odds ratio of 2.3. However, it has never been shown that an exhaustive workup for malignancy is cost-effective or beneficial. On the basis of a recent large cohort study, it has been recommended that the evaluation of idiopathic DVT be limited to a careful history, physical examination, CXR, routine blood counts and chemistries, FOBT, and possibly PSA in men and pelvic ultrasound in women. Further studies should be directed by this initial evaluation; if it is unrevealing, then additional tests will not likely help and may produce substantial psychological stress in the patient. (Answer: A—Careful history, physical examination, routine blood counts and chemistries, chest x-ray [CXR], fecal occult blood testing [FOBT], and prostate-specific antigen [PSA]; if these are not revealing, no further evaluation is necessary)

For more information, see Leung LLK: 5 Hematology: XIV Thrombotic Disorders. ACP Medicine Online (www.acpmedicine.com). Dale DC, Federman DD, Eds. WebMD Inc., New York, January 2005